Harr: Bacte

Ace your homework & exams now with Quizwiz!

B Nocardia spp. should be suspected if colonies that are partially acid fast by the traditional method are positive with the modified acid-fast method using Kinyoun stain and 1% sulfuric acid as the decolorizing agent. The other organisms listed are negative for acid-fast stain.

10. A gram-positive branching filamentous organism recovered from a sputum specimen was found to be positive with a modified acid-fast stain method. What is the most likely presumptive identification? A. Bacillus spp. B. Nocardia spp. C. Corynebacterium spp. D. Listeria spp.

C Putrescine is the amine product of the decarboxylation of ornithine.

Putrescine is an alkaline amine product of which bacterial enzyme? A. Arginine decarboxylase B. Phenylalanine deaminase C. Ornithine decarboxylase D. Lysine decarboxylase

C NFB will grow on the slant of TSI or KIA but they do not acidify the butt (glucose fermentation), as do the Enterobacteriaceae. NFB can be cytochrome oxidase positive or negative, but all the Enterobacteriaceae are oxidase negative. The Enterobacteriaceae grow well on MacConkey agar and reduce nitrate to nitrite, but the NFB grow poorly or not at all and most do not reduce nitrate. Nearly 70% of the NFB recovered from clinical specimens are: Strains of Psuedomonas aeruginosa Acinetobacter baumannii Stenotrophomonas maltophilia

What are the most appropriate screening tests to presumptively differentiate and identify the nonfermentative gram-negative bacilli (NFB) from the Enterobacteriaceae? A. Catalase, decarboxylation of arginine, growth on blood agar B. Motility, urease, morphology on blood agar C. Oxidase, TSI, nitrate reduction, growth on MacConkey agar D. Oxidase, indole, and growth on blood agar

A (SPS is used in most commercial blood culture products because it functions as an anticoagulant and prevents phagocytosis and complement activation. In addition, SPS neutralizes aminoglycoside antibiotics. Addition of SPS may inhibit some Neisseria and Peptostreptococcus, but this can be reversed with 1.2% gelatin.)

What is the purpose of adding 0.025%-0.050% sodium polyanetholsulfonate (SPS) to nutrient broth media for the collection of blood cultures? A. It inhibits phagocytosis and complement B. It promotes formation of a blood clot C. It enhances growth of anaerobes D. It functions as a preservative

C (The iodine should remain on the skin for 1 min because instant antisepsis does not occur when cleansing the skin for a blood culture.)

When cleansing the skin with alcohol and then iodine for the collection of a blood culture, the iodine (or iodophor) should remain intact on the skin for at least: A. 10 sec B. 30 sec C. 60 sec D. 5 min

B P. aeruginosa is often recovered from the respiratory secretions of cystic fibrosis patients. If the patient is chronically infected with the mucoid strain of P. aeruginosa, the biochemical identification is very difficult. The mucoid strain results from production of large amounts of alginate, a polysaccharide that surrounds the cell.

Which Pseudomonas is usually associated with a lung infection related to cystic fibrosis? A. P. fluorescens B. P. aeruginosa C. P. putida D. Burkholderia pseudomallei

C Cefsulodin-irgasan-novobiocin (CIN) medium is the best agar for the isolation of Yersinia strains because it inhibits growth of other coliforms, but it is not used routinely in clinical laboratories. Yersinia spp. grow well on MacConkey agar incubated at 37°C, but the colonies are much smaller than the other Enterobacteriaceae; therefore, 25°C is the temperature recommended for isolation. Some serotypes of Yersinia may be inhibited on more selective media, such as Salmonella-Shigella or Hektoen. CNA agar inhibits the growth of gram-negative bacteria.

Which agar that is used for routine stool cultures is the medium of choice for the isolation of Yersinia strains from stool specimens? A. Salmonella-Shigella agar B. Hektoen enteric agar C. MacConkey agar D. CNA agar

A Both organisms produce yellowish pigment and have polar tuft flagella, but the oxidase and DNase tests are differential.

Which biochemical tests are needed to differentiate Burkholderia cepacia from S. maltophilia? A. Pigment on blood agar, oxidase, DNase B. Pigment on MacConkey agar, flagellar stain, motility C. Glucose, maltose, lysine decarboxylase D. TSI, motility, oxidase

D (The aminoglycoside antibiotics are bactericidal agents that act by inhibiting protein synthesis. They show a low incidence of bacterial resistance but must be monitored carefully because at high doses they can cause ototoxicity and nephrotoxicity. The group includes amikacin, gentamicin, tobramycin, kanamycin, streptomycin, and spectinomycin. These drugs are usually administered intravenously or intramuscularly because they are poorly absorbed from the gastrointestinal tract.)

Which class of antibiotics is used for the treatment of serious gram-negative infections as well as infections with Mycobacterium tuberculosis? A. Cephalosporins B. Penicillins C. Tetracyclines D. Aminoglycosides

A Yersinia pestis is the cause of bubonic and pneumonic plague. Bubonic plague causes swelling of the groin lymph nodes (bubos), whereas pneumonic plague involves the lungs. The infection caused by bubonic plague may result in fulminant bacteremia that is usually fatal. The transmission is from rodents (rats, ground squirrels, or prairie dogs) to humans by the bite of fleas (vectors) or by ingestion of contaminated animal tissues. Pneumonic plague is acquired via the airborne route when there is close contact with other pneumonic plague victims.

Following a 2-week camping trip to the Southwest (US), a 65-year-old male patient was hospitalized with a high fever and an inflammatory swelling of the axilla and groin lymph nodes. Several blood cultures were obtained, resulting in growth of gram-negative rods resembling "closed safety pins." The organism grew on MacConkey's agar showing non-lactose-fermenting colonies. Testing demonstrated a nonmotile rod that was biochemically inert. What is the most likely identification? A. Yersinia pestis B. Klebsiella pneumoniae C. Proteus vulgaris D. Morganella morganii

A Methicillin-resistant S. aureus (MRSA) and methicillin-resistant S. epidermidis (MRSE) are called heteroresistant. This refers to two subpopulations in a culture, one that is susceptible and the other that is resistant to antibiotic(s). The resistant population grows more slowly than the susceptible one and can be overlooked. Therefore, the more resistant subpopulation should be promoted growthwise by using a neutral pH (7.0-7.4), cooler incubation temperatures (30°C-35°C), the addition of 2%-4% NaCl, and incubation up to 48 hours.

10. Strains of Staphylococcus species resistant to the β-lactam antibiotics by standardized disk diffusion and broth microdilution susceptibility methods are called: A. Heteroresistant B. Bacteriophage group 52A C. Cross resistant D. Plasmid altered

A Infant botulism is the most frequent form occurring in the United States. Epidemiological studies have demonstrated that infant botulism results from the ingestion of spores via breastfeeding or exposure to honey. Preformed toxin has not been detected in food or liquids taken by the infants. C. botulinum multiplies in the gut of the infant and produces the neurotoxin in situ.

12. Which mechanism is responsible for botulism in infants caused by Clostridium botulinum? A. Ingestion of spores in food or liquid B. Ingestion of preformed toxin in food C. Virulence of the organism D. Lipase activity of the organism

B Burkholderia pseudomallei infections often produce abscesses in organs (liver, spleen, lungs) as well as on the skin, in soft tissue, and in joints and bones. Vietnam War veterans especially may harbor these organisms, which are limited to tropical and subtropical environments (Southeast Asia and Australia). The organism may surface years later after surviving in a latent state within phagocytes. The surfaces of rice paddies in Northern Thailand have a high prevalence of this organism.

24. A Vietnam War veteran presented with a "glanders-like" infection (melioidosis). Several blood cultures produced gram-negative rods that were positive for cytochrome oxidase, oxidized glucose and xylose, and grew at 42°C. What is the most likely organism? A. Stenotrophomonas maltophilia B. Burkholderia pseudomallei C. Pseudomonas aeruginosa D. Acinetobacter spp.

B P. aeruginosa is an opportunistic organism that is not part of the human normal flora. Contact lens solution contamination, eye injury, or contact lens eye trauma are factors that contribute to P. aeruginosa eye infections. The characteristic blue-green pigment on Mueller-Hinton agar (pyocyanin pigment) produced by P. aeruginosa and the high resistance to antibiotics aid in its identification.

A 15-year-old female complained of a severe eye irritation after removing her soft-contact lenses. A swab of the infected right eye was obtained by an ophthalmologist, who ordered a culture and sensitivity test. The culture was plated on blood agar and MacConkey agar. At 24 hours, growth of a gram-negative rod that tested positive for cytochrome oxidase was noted. The Mueller-Hinton sensitivity plate showed a bluish-green "lawn" of growth that proved highly resistant to most of the antibiotics tested except amikacin, tobramycin, and ciprofloxacin. What is the most likely identification? A. Burkholderia cepacia B. Pseudomonas aeruginosa C. Stenotrophomonas maltophilia D. Acinetobacter baumannii

A Burkholderia mallei is rarely transmitted to humans. It is the causative agent of glanders in mules, donkeys, and horses. It is not part of the human skin flora and the most likely transmission to humans is through broken skin.

A 20-year-old horse groomer exhibited a "glanders-like" infection. His history indicated he had suffered several open wounds on his hands 2 weeks before the swelling of his lymph nodes. A gram-negative rod was recovered from a blood culture that grew well on blood and MacConkey agars. Most of the biochemical tests were negative, including the cytochrome oxidase test. What is the most likely identification? A. Burkholderia mallei B. Pseudomonas aeruginosa C. Pseudomonas stutzeri D. Burkholderia pseudomallei

D Persons handling samples suspected of containing F. tularensis must wear gloves and use a biological safety cabinet (follow biosafety Level II controls). For cultures, biosafety Level III controls must be followed. Tularemia is one of the most common laboratory-acquired infections, and it is recommended that specimens be sent to a reference laboratory for identification and serological testing. F. tularensis requires cysteine and cystine to grow. It may grow on chocolate agar supplemented with IsoVitalex and also on charcoal yeast extract agar used to isolate Legionellae.

A 29-year-old male who often hunted rabbits and spent a lot of time in the woods was admitted to the hospital with skin ulcers on his upper extremities. At 48 hours, a small coccobacillus was recovered from the aerobic blood culture bottle only. The organism stained poorly with Gram stain, but did stain with acridine orange. Cultures taken from the ulcers did not grow on primary media. What is the most likely identification? A. Pseudomonas aeruginosa B. Pseudomonas fluorescens C. Chryseobacterium spp. D. Francisella tularensis

A C. canimorsus are part of the oral flora of dogs. The organisms require at least 5% CO2 for growth and grow slowly on blood and chocolate agars. Colonies can grow in 48 hours if cultured in high CO2 on BHI agar with 5% sheep blood.

A 46-year-old dog warden was admitted to the hospital with several puncture bite wounds encountered while wrangling with a stray dog. Culture at 48 hours produced small yellow colonies on 5% sheep blood and chocolate agars in 10% CO2, but no growth on MacConkey agar. Gram stain showed gram-negative curved, fusiform rods. Colonies were oxidase and catalase positive. What is the most likely identification? A. Capnocytophaga canimorsus B. Francisella tularensis C. Legionella pneumophila D. Pseudomonas aeruginosa

B B. pertussis, the cause of whooping cough, is highly contagious during the 5-10 day period after acquisition. The incidence of whooping cough is greater in nonimmunized individuals, and therefore, is higher in children under 1 year of age. B. bronchiseptica is only rarely found in humans, but may cause respiratory disease in animals. Unlike B. pertussis it is positive for nitrite, urease, and motility.

A 5-year-old nonimmunized male with a persistent cough, fever, and flulike symptoms was admitted to the hospital. Nasopharyngeal swabs were cultured on 15% blood, chocolate, Bordet-Genjou, and Regan-Lowe (with 10% charcoal) agars. All media grew a gram-negative coccobacillus. Carbohydrate and biochemical tests were negative. What is the most likely identification? A. Haemophilus influenza B. Bordetella pertussis C. Haemophilus parainfluenzae D. Bordetella bronchiseptica

D Corynebacterium species recovered from a throat culture are usually considered part of the normal throat flora. C. diphtheriae is an exception and should be suspected when one of the conditions described occurs. In this event, direct inoculation on Loeffler serum medium or tellurite medium and the following biochemical tests should be performed to confirm the identification of C. diphtheriae. Gelatin hydrolysis = Neg Catalase = + Motility = + Urease = + Acid from glucose = + Carbohydrate fermentation = +

A Corynebacterium species recovered from a throat culture is considered a pathogen when it produces: A. A pseudomembrane of the oropharynx B. An exotoxin C. Gray-black colonies with a brown halo on Tinsdale's agar D. All of these options

A A Gram stain of urethral discharge (in men only) showing typical gonococcal cells in PMNs should be reported "presumptive N. gonorrhoeae, confirmation to follow." With female patients, the normal vaginal flora contain gram-negative cocci and diplococci resembling gonococci and, therefore, no presumptive identification should be reported for N. gonorrhoeae from the vaginal Gram stain smear.

A Gram stain of a urethral discharge from a man showing extracellular and intracellular gram-negative diplococci within segmented neutrophils is a presumptive identification for: A. Neisseria gonorrhoeae B. Neisseria meningitidis C. Moraxella (Branhamella) catarrhalis D. Neisseria lactamica

C With the exception of M. tuberculosis, M. avium-intracellulare (MAI) complex is the Mycobacterium species most often isolated from AIDS patients. It is biochemically inert, which is a distinguishing factor for identification. MAI complex is highly resistant to the antibiotics used to treat tuberculosis, including multidrug therapy. Treatment with streptomycin, rifampin, ethionamide, ethambutol with cycloserine, or kanamycin has shown little success.

A Mycobacterium species recovered from a patient with AIDS gave the following results: Niacin = Neg T2H = + Tween 80 hydrolysis = Neg Nitrate reduction = Neg Heat-stable catalase (68°C) = ± Nonphotochromogen What is the most likely identification? A. M. gordonae B. M. bovis C. M. avium-intracellulare complex D. M. kansasii

D S. aureus is novobiocin sensitive and cannot be ruled out by a negative clumping factor test. Most S. aureus produce β-hemolysis on sheep blood agar plates and are mannitol salt positive (produce acid and are not inhibited by the high salt concentration). The tube test should be performed because the slide test was negative.

A Staphylococcus spp. recovered from a wound (cellulitis) was negative for the slide coagulase test (clumping factor) and negative for novobiocin resistance. The next test(s) needed for identification is (are): A. Tube coagulase test B. β-Hemolysis on blood agar C. Mannitol salt agar plate D. All of these options

B V. parahaemolyticus appear as green colonies on TCBS agar, whereas V. cholerae appear as yellow colonies on TCBS. V. cholerae is the only Vibrio species that causes a positive string test. In the test, a loopful of bacterial colonies is suspended in sodium deoxycholate, 0.5%, on a glass slide. After 60 seconds, the inoculating loop is lifted out of the suspension. V. cholerae forms a long string resembling a string of pearls. Salmonella spp. and Shigella spp. are oxidase negative.

A curved gram-negative rod producing oxidase-positive colonies on blood agar was recovered from a stool culture. Given the following results, what is the most likely identification? Lysine decarboxylase = + Arginine decarboxylase = Neg Indole = + KIA = Alk/Acid VP = Neg Lactose = Neg Urease = ± String test = Neg TCBS agar = Green colonies A. Vibrio cholerae B. Vibrio parahaemolyticus C. Shigella spp. D. Salmonella spp.

A Corynebacterium spp. are part of the normal upper respiratory tract flora. Organisms display typical pleomorphic shapes often resembling letters such as Y or L, and metachromatic granules. Identification of C. diphtheriae, however, requires selective culture media and biochemical testing.

A direct smear from a nasopharyngeal swab stained with Loeffler methylene blue stain showed various letter shapes and deep blue, metachromatic granules. The most likely identification is: A. Corynebacterium spp. B. Nocardia spp. C. Listeria spp. D. Gardnerella spp.

A The CAMP (hemolytic phenomenon first described by Christie, Atkins, and Munch-Petersen in 1944) test refers to a hemolytic interaction that is seen on a blood agar plate between the β-hemolysins produced by most strains of S. aureus and an extracellular protein produced by both hemolytic and nonhemolytic isolates of group B streptococci. When performing a CAMP test, the plate must be placed in an ambient air incubator at 35°C-37°C. Group A streptococci may be CAMP positive if the plate is incubated in a candle jar, high CO2 atmosphere, or anaerobically.

A false-positive CAMP test for the presumptive identification of group B streptococci may occur if the plate is incubated in a(n): A. Candle jar or CO2 incubator B. Ambient air incubator C. 35°C incubator D. 37°C incubator

A The antistreptolysin O (ASO) titer is used to indicate a recent infection with group A β-hemolytic streptococci. Streptolysin O may also be produced by some strains of groups C and G streptococci.

A fourfold rise in titer of which antibody is the best indicator of a recent infection with group A β-hemolytic streptococci? A. Anti-streptolysin O B. Anti-streptolysin S C. Anti-A D. Anti-B

B The only Campylobacter spp. that hydrolyze hippurate are C. jejuni and subsp. doylei. However, some strains of P. aeruginosa grow on agar selective for Campylobacter at 42°C. C. fetus will not grow at 42°C but will grow at 25°C and 37°C.

A gram-negative S-shaped rod recovered from selective media for Campylobacter species gave the following results: Catalase = + Oxidase = + Motility = + Hippurate hydrolysis = + Growth at 42°C = + Nalidixic acid = Susceptible Pigment = Neg Grape odor = Neg Cephalothin = Resistant The most likely identification is: A. Pseudomonas aeruginosa B. Campylobacter jejuni C. Campylobacter fetus D. Pseudomonas putida

A Lactobacillus spp. produce both long, slender rods or short coccobacilli that form chains. Lactobacillus spp. are part of the normal flora of the vagina (are not considered a pathogen) and are sometimes confused with the streptococci.

A non-spore-forming, slender gram-positive rod forming palisades and chains was recovered from a vaginal culture and grew well on tomato juice agar. The most likely identification is: A. Lactobacillus spp. B. Bacillus spp. C. Neisseria spp. D. Streptococcus spp.

D All of the listed organisms produce mycelium (aerial or substrate), causing them to appear branched when Gram stained, but only the Nocardia spp. are positive for modified acid-fast stain. Nocardia is an opportunistic pathogen, and cultures typically have a musty basement odor.

A gram-positive (gram-variable), beaded organism with delicate branching was recovered from the sputum of a 20-year-old patient with leukemia. The specimen produced orange, glabrous, waxy colonies on Middlebrook's agar that showed partial acid-fast staining with the modified Kinyoun stain. What is the most likely identification? A. Rhodococcus spp. B. Actinomadura spp. C. Streptomyces spp. D. Nocardia spp.

C Micrococcus spp. utilize glucose oxidatively but not under anaerobic conditions (sealed tube). Staphylococcus spp. utilize glucose oxidatively and anaerobically. The catalase differentiates the Micrococcaceae family (positive) from the Streptococcaceae family (negative).

A gram-positive coccus recovered from a wound ulcer from a 31-year-old diabetic patient showed pale yellow, creamy, β-hemolytic colonies on blood agar. Given the following test results, what is the most likely identification? Catalase = + Glucose OF: positive open tube, negative sealed tube Mannitol salt = Neg Slide coagulase = Neg A. Staphylococcus aureus B. Staphylococcus epidermidis C. Micrococcus spp. D. Streptococcus spp.

C C. perfringens produces a double zone of β-hemolysis on blood agar, which makes identification relatively easy. The inner zone of complete hemolysis is caused by a θ-toxin and the outer zone of incomplete hemolysis is caused by an α-toxin (lecithinase activity). The Bacteroides spp. are gram-negative bacilli, and C. difficile is lecithinase negative and does not produce a double zone of β-hemolysis.

A gram-positive spore-forming bacillus growing on sheep-blood agar anaerobically produces a double zone of β-hemolysis and is positive for lecithinase. What is the presumptive identification? A. Bacteroides ureolyticus B. Bacteroides fragilis C. Clostridium perfringens D. Clostridium difficile

A The American Thoracic Society recognizes three levels of laboratory services for mycobacteria testing. Level I laboratories are those that grow mycobacteria and perform acid-fast stains but do not identify M. tuberculosis (they may or may not perform drug susceptibility tests on M. tuberculosis). Level II laboratories perform all of the functions of Level I laboratories and also identify M. tuberculosis. Level III laboratories identify all mycobacteria species from clinical specimens and perform drug susceptibility tests on all species.

A laboratory provides the following services for identification of mycobacteria: Acid-fast staining of clinical specimens Inoculation of cultures Shipment of positive cultures to a reference laboratory for identification According to the American Thoracic Society's definition for levels of service this laboratory is: A. Level I B. Level II C. Level III D. Level IV

A E. corrodens is a part of the normal flora of the upper respiratory tract and the mouth. It is often seen after trauma to the head and neck, dental infections, and human bite wounds. It requires blood for growth. The organism causes a pitting of the agar where colonies are located. The smell of bleach may be apparent when the plates are uncovered for examination. Actinobacillus spp. and C. hominis both utilize several carbohydrates, and Proteus spp. are oxidase negative.

A mixture of slender gram-negative rods and coccobacilli with rounded ends was recovered from blood cultures following a patient's root canal surgery. Given the following results after 48 hours, what is the most likely organism? Catalase = Neg Ornithine decarboxylase = + Urease = Neg Lysine decarboxylase = + Oxidase = + X and V requirement = Neg Indole = Neg Carbohydrates = Neg (no acid produced) Growth on blood and chocolate agar = + (with pitting of agar) Growth on MacConkey agar = Neg A. Eikenella corrodens B. Actinobacillus spp. C. Cardiobacterium hominis D. Proteus spp.

B Chryseobacterium meningosepticum can cause septicemia and meningitis in neonates and immunocompromised adults. The ability to encapsulate, produce proteases, and survive in chlorinated tap water are factors that contribute to hospital-acquired infections with this bacterium.

A neonate was readmitted to the hospital with a diagnosis of meningitis. The CSF revealed gram-negative straight rods. At 24 hours, the organism grew on 5% sheep blood and chocolate agars displaying a yellow pigment. On MacConkey agar, it appeared as a non-lactose fermenter. Colonies were oxidase, DNase, and gelatinase positive, and oxidized glucose and mannitol. What is the most likely identification? A. Haemophilus influenza B. Chryseobacterium meningosepticum C. Stenotrophomonas maltophilia D. Acinetobacter baumannii

D A Gram stain smear from a vaginal secretion showing many squamous epithelial cells loaded with pleomorphic gram-variable (positive and negative) bacilli is considered presumptive for G. vaginalis. Such cells are called clue cells. Other important findings are: β-Hemolysis on BAP = + Catalase = Neg Oxidase = Neg Hippurate hydrolysis = +

A presumptive diagnosis of Gardnerella vaginalis can be made using which of the following findings? A. Oxidase and catalase tests B. Pleomorphic bacilli heavily colonized on vaginal epithelium C. Hippurate hydrolysis test D. All of these options

D The β-hemolytic streptococci—not of groups A, B, or D—are sensitive to SXT and may be either sensitive or resistant to bacitracin. Groups A and B are both resistant to SXT. Group A and Enterococcus faecalis are PYR positive. Enterococcus faecalis is also positive for bile esculin and 6.5% salt broth.

A pure culture of β-hemolytic streptococci recovered from a leg wound ulcer gave the following reactions: CAMP test = Neg Hippurate hydrolysis = Neg Bile esculin = Neg 6.5% salt = Neg PYR = Neg Bacitracin = Resistant Optochin = Resistant SXT = Sensitive The most likely identification is: A. Group A streptococci B. Group B streptococci C. Enterococcus faecalis D. Nongroup A, nongroup B, nongroup D streptococci

B Although several biotypes of H. parainfluenzae produce indole and urease, H. parainfluenzae does not require X factor for growth. H. ducreyi requires X factor but not V factor. H. aphrophilus does not require either X factor or V factor for growth.

A small, gram-negative coccobacillus recovered from the CSF of a 2-year-old child gave the following results: Indole = + Glucose = + (acid) X requirement = + V requirement = + Urease = + Lactose = Neg Sucrose = Neg Hemolysis = Neg Which is the most likely identification? A. Haemophilus parainfluenzae B. Haemophilus influenzae C. Haemophilus ducreyi D. Haemophilus aphrophilus

A The best differentiating test to perform on a suspected B. anthracis culture is the 10-unit penicillin disk test. B. anthracis is susceptible but other Bacillus spp. are not. Organisms suspected to be B. anthracis should be sent to a reference laboratory for final confirmation. All tests should be performed in a biological safety hood, and personnel should wear protective clothing to reduce risk from possible production of aerosols.

A suspected Bacillus anthracis culture obtained from a wound specimen produced colonies that had many outgrowths (Medusa-head appearance), but were not β-hemolytic on sheep blood agar. Which test should be performed next? A. Penicillin (10-unit) susceptibility test B. Lecithinase test C. Glucose test D. Motility test

D Legionella spp. stain poorly if at all with Gram stain. Legionella pneumophilia is not acid fast although L. micdadei, which accounts for a small percentage of Legionella pneumonia infections, is acid-fast positive. Specimens suspected of containing Legionella spp. should be handled in a Class II biological safety cabinet. Legionella spp. require buffered-charcoal-yeast extract (BCYE) agar for growth and will not grow on MacConkey agar. Since culture can take up to 10 days, rapid diagnosis by direct immunofluorescence and DNA amplification are preferred. Direct fluorescent antibody tests are not as sensitive as culture or PCR, but are specific and can be used to rapidly confirm a positive serological test, which may be positive in the absence of disease.

A suspected case of Legionnaires' disease was noted on the request form for a culture and sensitivity ordered on a sputum sample. The patient was a 70-year-old male who presented with a positive serological test for Legionella spp. What is the most efficient way to confirm the infection using the submitted sample? A. Culture the sputum on MacConkey agar B. Gram stain of the sputum C. Acid-fast staining D. Direct immunofluorescent microscopy

C The growth of yellow or green colonies on the selective TCBS agar (thiosulfate citrate bile salts sucrose) is dependent on whether the organism ferments sucrose (producing yellow colonies). Vibrio also grow well on 5% sheep blood, chocolate, and MacConkey agars. Enrichment with alkaline peptone broth, pH 8.4, helps in recovering Vibrio spp. from stool specimens.

A visitor to South America who returned with diarrhea is suspected of being infected with V. cholerae. Select the best medium for recovery and identification of this organism. A. MacConkey agar B. Blood agar C. TCBS agar D. XLD agar

B The CAP lists four options for laboratories to follow in order to correlate the services provided with guidelines for inspection and accreditation. A laboratory's performance on CAP proficiency tests is evaluated by interlaboratory comparison with laboratories within these levels of performance.

According to the College of American Pathologists (CAP) guidelines, which services for mycobacteria would be performed by a Level II laboratory? A. No procedures performed B. Acid-fast staining, inoculation, and referral to a reference laboratory C. Isolation and identification of Mycobacterium tuberculosis; preliminary identification of other species D. Definitive identification of all mycobacteria

A Acid-fast smears are standardized by the American Thoracic Society for reporting the number of AFB seen. The following criteria should be used to uniformly report results: 1-2 AFB per smear: Report number seen and request another sample 3-9 AFB per smear: Report as rare (1+) 10 or more per smear: Report as few (2+) 1-9 or more per oil immersion field: Report as numerous (3+)

According to the reporting standards of the American Thoracic Society, one or more acid-fast bacilli (AFB) per oil immersion field (1,000×) are reported as: A. Numerous or 3+ B. Few or 2+ C. Rare or 1+ D. Indeterminate; a new specimen should be requested

A M. tuberculosis is positive for niacin accumulation, while the other three species are niacin negative. M. ulcerans is associated with skin infections (in the tropics), does not grow at 37°C (optimal temperature is 33°C), and is not recovered from sputum. A serpentine pattern of growth indicates production of cording factor, a virulence factor for M. tuberculosis.

Acid-fast staining of a smear prepared from digested sputum showed slender, slightly curved, beaded, red mycobacterial rods. Growth on Middlebrook 7H10 slants produced buff-colored microcolonies with a serpentine pattern after 14 days at 37°C. Niacin and nitrate reduction tests were positive. What is the most probable presumptive identification? A. Mycobacterium tuberculosis B. Mycobacterium ulcerans C. Mycobacterium kansasii D. Mycobacterium avium-intracellulare complex

B Niacin production is common to all mycobacteria. However, the niacin accumulates as a water- soluble metabolite in the culture medium when the organism cannot form niacin ribonucleotide. M. tuberculosis, M. simiae, and some strains of M. marinum, M. chelonae, and M. bovis lack the enzyme and therefore are called niacin positive because of the accumulation of niacin detected in the test medium.

All of the following Mycobacterium spp. produce the enzyme required to convert niacin to niacin ribonucleotide except: A. M. kansasii B. M. tuberculosis C. M. avium-intracellulare complex D. M. szulgai

D Veillonella spp. are gram-negative cocci. All four genera are part of the normal human flora and are the anaerobic cocci most frequently isolated from blood cultures, abscesses, wounds, and body fluids. The Streptococcus spp. are facultative anaerobes, but only Streptococcus intermedius is classified as an obligate anaerobe.

All of the following genera are anaerobic cocci that stain gram positive except: A. Peptococcus spp. B. Peptostreptococcus spp. C. Streptococcus spp. D. Veillonella spp.

D M. kansasii is a photochromogen that causes chronic pulmonary disease (classic tuberculosis). The other three species cause cutaneous or subcutaneous disease. It is important to culture skin lesions at the correct temperature to facilitate growth.

All of the following mycobacteria are associated with skin infections except: A. Mycobacterium marinum B. Mycobacterium haemophilum C. Mycobacterium ulcerans D. Mycobacterium kansasii

A Bartonella spp. are difficult to grow on primary culture media. When CSD is suspected from the patient's history, blood cultures should be smeared and Gram stained. Bartonella spp. are biochemically inert, meaning that they are negative for oxidase, catalase, indole, and urease tests. Therefore, commercial identification systems, DNA amplification for various genes, and indirect immunofluorescence assays are used to identify these organisms.

An elderly woman who cared for several domestic cats was hospitalized with suspected cat-scratch disease (CSD). Blood cultures appeared negative, but a small, slightly curved pleomorphic gram-negative bacillus grew on BHI agar (brain, heart infusion agar with 5% horse or rabbit blood). What is the most likely identification? A. Bartonella spp. B. Brucella spp. C. Kingella spp. D. Haemophilus spp.

D The standard Kirby-Bauer method used for disk diffusion susceptibility testing recommended by CLSI is ≥ 20 mm. The test requires 20-24-hour incubation at 35°C in 5%-7% CO2. If the zone size is ≤ 19 mm, an MIC should be performed on isolates from CSF or blood.

An oxacillin-disk screen test is used to detect Streptococcus pneumonia resistance to penicillin. Using Mueller-Hinton agar with 5% sheep blood and a 1 μg oxacillin disk, what is the recommended inhibition zone size for penicillin susceptibility? A. ≥5 mm B. ≥10 mm C. ≥15 mm D. ≥20 mm

D The incidence of anaerobic bacteria recovered from the urine is approximately 1% of isolates. The other three types of infection are associated with a 60%-93% incidence of anaerobic recovery. Urine is not cultured routinely under anaerobic conditions unless obtained surgically (e.g., suprapubic aspiration).

Anaerobic bacteria are routinely isolated from all of the following types of infections except: A. Lung abscesses B. Brain abscesses C. Dental infections D. Urinary tract infections

C The anaerobes are not suited for the broth disk elution or disk agar diffusion tests because of their slow rate of growth. Kirby-Bauer method reference charts are not designed to be used as a reference of susceptibility for anaerobes.

Antimicrobial susceptibility testing of anaerobes is done by which of the following methods? A. Broth disk elution B. Disk agar diffusion C. Microtube broth dilution D. β-Lactamase testing

D Both MR and VP tests detect acid production from the fermentation of glucose. However, a positive MR test denotes a more complete catabolism of glucose to highly acidic end products such as formate and acetate than occurs with organisms that are VP positive only (e.g., Klebsiella pneumoniae).

At which pH does the methyl red (MR) test become positive? A. 7.0 B. 6.5 C. 6.0 D. 4.5

A Both species of Bacillus are catalase and lecithinase positive and produce acid from glucose. B. cereus is β-hemolytic and motile, but B. anthracis is neither. See the following chart.

Bacillus anthracis and Bacillus cereus can best be differentiated by which tests? A. Motility and β-hemolysis on a blood agar plate B. Oxidase and β-hemolysis on a blood agar plate C. Lecithinase and glucose D. Lecithinase and catalase

A The bacitracin disk test is used in conjunction with other confirmatory tests for the β-hemolytic streptococci. In addition to group A, groups C, F, and G are also β-hemolytic and give a positive test for bacitracin (a zone of inhibition of any size). Therefore, a positive test does not confirm the presence of group A β-hemolytic streptococci.

Bacitracin A disks (0.04 unit) are used for the presumptive identification of which group of β-hemolytic streptococci? A. Group A B. Group B C. Group C D. Group F

A A bacitracin disk (0.04 unit) is used to identify group A β-hemolytic streptococci, but it will also differentiate catalase-positive organisms. A zone of 10 mm or greater is considered susceptible. The Staphylococcus species are resistant and grow up to the disk, while Micrococcus species are sensitive.

Bacitracin resistance (0.04 unit) is used to differentiate: A. Micrococcus spp. from Staphylococcus spp. B. Staphylococcus spp. from Neisseria spp. C. Planococcus spp. from Micrococcus spp. D. Staphylococcus spp. from Streptococcus spp.

A The family Enterobacteriaceae consists of more than 100 species and represents the most commonly encountered isolates in clinical specimens. All Enterobacteriaceae ferment glucose and are oxidase negative and nonsporulating. Most Enterobacteriaceae are motile, but the genera Shigella and Klebsiella are not.

Biochemically, the Enterobacteriaceae are gram-negative rods that: A. Ferment glucose, reduce nitrate to nitrite, and are oxidase negative B. Ferment glucose, produce indophenol oxidase, and form gas C. Ferment lactose and reduce nitrite to nitrogen gas D. Ferment lactose and produce indophenol oxidase

D (CNA agar inhibits the growth of gram-negative bacteria and is used to isolate gram-positive cocci from specimens. This medium is especially useful for stool and wound cultures because these may contain large numbers of gram-negative rods.)

Colistin-nalidixic acid agar (CNA) is used primarily for the recovery of: A. Neisseria species B. Enterobacteriaceae C. Pseudomonas aeruginosa D. Staphylococcus aureus

A Serological confirmation of Shigella isolates is based upon O antigen typing. If a suspected Shigella spp. is serologically typed with polyvalent sera before it has been correctly identified biochemically, a false-positive confirmation may occur with an isolate that is E. coli (i.e., anaerogenic non-gas-producing, lactose-negative or delayed, and nonmotile strains). These strains were formerly known as the Alkalescens-Dispar serotype.

Care must be taken when identifying biochemical isolates of Shigella because serological cross reactions occur with: A. E. coli B. Salmonella spp. C. Pseudomonas spp. D. Proteus spp.

A B. cepacia (93%) are weakly oxidase positive and motile. Chryseobacterium spp. are oxidase positive but are nonmotile.

Chryseobacterium spp. and B. cepacia are easily differentiated by which test? A. Motility B. OF glucose C. Oxidase D. Cetrimide agar

C E. rhusiopathiae is catalase negative, whereas the other three organisms are catalase positive. E. rhusiopathiae is seen primarily as a skin infection on the fingers of meat and poultry workers. Colonies growing on blood agar are small and transparent, may be either smooth or rough, and are often surrounded by a green tinge. E. rhusiopathiae is characterized by H2S production *********** of a TSI slant, which differentiates it from other catalase-negative, gram-positive rods.

Culture of a finger wound specimen from a meat packer produced short gram-positive bacilli on a blood agar plate with no hemolysis. Given the following test results at 48 hours, what is the most likely identification? Catalase = Neg H2S/TSI = + Motility (wet prep) = Neg Motility (media) = Neg (bottle-brush growth in stab culture) A. Bacillus cereus B. Listeria monocytogenes C. Erysipelothrix rhusiopathiae D. Bacillus subtilis

A M. marinum is typically recovered from cutaneous wounds resulting from infection when the skin is traumatized and comes into contact with inadequately chlorinated fresh water or salt water, such as in swimming pools or fish aquariums. The other three species are slow growers at 37°C. M. tuberculosis and M. avium-intracellulare complex are nonphotochromogens. M. avium-intracellulare complex is urease negative, M. tuberculosis is positive for niacin and nitrate, and M. kansasii is positive for nitrate and catalase.

Culture of a skin (hand) wound from a manager of a tropical fish store grew on Löwenstein-Jensen agar slants at 30°C in 10 days but did not grow on the same media at 37°C in 20 days. Given the following results, what is the most likely identification? Photochromogen = + Niacin = Neg Urease = + Heat-stable catalase (68°C) = Neg Nitrate reduction = Neg Tween 80 hydrolysis = + A. Mycobacterium marinum B. Mycobacterium kansasii C. Mycobacterium avium-intracellulare complex D. Mycobacterium tuberculosis

D N. lactamica is part of the normal vaginal and throat flora and is the only Neisseria species that grows on MTM that utilizes lactose. Other saprophytic Neisseria spp. may utilize lactose but do not grow on MTM media.

Culture on MTM media of a vaginal swab produced several colonies of gram-negative diplococci that were catalase and oxidase positive and Superoxol negative. Given the following carbohydrate reactions, select the most likely identification. Glucose = + Sucrose = Neg Lactose = + Maltose = + Fructose = Neg A. Neisseria gonorrhoeae B. Neisseria sicca C. Neisseria flavescens D. Neisseria lactamica

C Capnocytophaga gingivalis, C. sputigena, and C. ochracea are part of the normal oropharyngeal flora of humans; however, C. canimorsus and C. cynodegmi (formerly CDC groups DF-2 and DF-2-like bacteria) are associated with infections resulting from dog bite wounds.

Cultures obtained from a dog bite wound produced yellow, tan, and slightly pink colonies on blood and chocolate agar with a margin of fingerlike projections appearing as a film around the colonies. Given the following results at 24 hours, which is the most likely organism? Oxidase = + Catalase = + Growth on MacConkey agar = Neg Motility = Neg A. Actinobacillus spp. B. Eikenella spp. C. Capnocytophaga spp. D. Pseudomonas spp.

D (CCFA is used for recovery of C. difficile from stool cultures. Cycloserine and cefoxitin inhibit growth of gram-negative coliforms in the stool specimen. C. difficile ferments fructose, forming acid that, in the presence of neutral red, causes the colonies to become yellow.)

Cycloserine-cefoxitin-fructose agar (CCFA) is used for the recovery of: A. Yersinia enterocolitica B. Yersinia intermedia C. Clostridium perfringens D. Clostridium difficile

C CTA agar with 1% carbohydrate and phenol red pH indicator added is used for the identification of Neisseria species. CTA carbohydrates must be placed in an ambient air incubator because a high CO2 concentration may reduce the pH, causing a false-positive (acid) result. The utilization of carbohydrates by some fastidious gonococcal strains may take up to 72 hours in order to produce a color change in the pH indicator.

Cystine tryptic digest (CTA) media used for identification of Neisseria spp. should be inoculated and cultured in: A. A CO2 incubator at 35°C for 24 hours B. A CO2 incubator at 42°C for up to 72 hours C. A nonCO2 incubator at 35°C for up to 72 hours D. An anaerobic incubator at 35°C for up to 72 hours

D Infected CF patients usually do not escape P. aeruginosa infections completely. P. aeruginosa produces alginate that accounts for the "wet, mucoidy" appearance of colonies. This overproduction of alginate is thought to cause the inhibition of phagocytosis. The result is chronic infections in CF patients with the "wet" form of P. aeruginosa.

Cytochrome oxidase-positive, nonfermentative gram-negative bacilli were recovered from the stool of a cystic fibrosis (CF) patient. The isolates produced wet (mucoidy) colonies on blood agar. Which identification is most likely? A. Acinetobacter spp. B. Pseudomonas alcaligenes C. Pseudomonas stutzeri D. Pseudomonas aeruginosa

D Specific decarboxylases split dibasic amino acids (lysine, arginine, and ornithine), forming alkaline amines. These products turn the pH indicators in the medium (cresol red and bromcresol purple) from yellow to purple.

Decarboxylation of the amino acids lysine, ornithine, and arginine results in the formation of: A. Ammonia B. Urea C. CO2 D. Amines

A Egg yolk agar (modified McClung's or neomycin egg yolk agar) is used to determine the presence of lecithinase activity, which causes an insoluble, opaque, whitish precipitate within the agar. Lipase activity is indicated by an iridescent sheen or pearly layer on the surface of the agar.

Egg yolk agar is used to detect which enzyme produced by Clostridium species? A. Lecithinase B. β-Lactamase C. Catalase D. Oxidase

A E. sakazakii is called a yellow-pigmented E. cloacae and is best differentiated from E. cloacae by sorbitol fermentation (95% positive for E. cloacae and 0% for E. sakazakii). In addition, E. cloacae is usually positive for urease and malonate (65% and 75%, respectively) and E. sakazakii is usually negative (1% and < 20%, respectively). Both species are usually motile and arginine dihydrolase positive.

Enterobacter sakazakii can best be differentiated from Enterobacter cloacae by which of the following characteristics? A. Yellow pigmentation and negative sorbitol fermentation B. Pink pigmentation and positive arginine dihydrolase C. Yellow pigmentation and positive urease D. H2S production on TSI

A Shigella spp. and Campylobacter spp. are both causes of diarrhea, abdominal pain, fever, and sometimes vomiting. Blood is present in the stools of patients infected with Shigella as a result of invasion and penetration of the bowel. Young children may also exhibit bloody stools when infected with Campylobacter.

Fever, abdominal cramping, watery stools, and fluid and electrolyte loss preceded by bloody stools 2-3 days before is characteristic of shigellosis but may also result from infection with: A. Campylobacter spp. B. Salmonella spp. C. Proteus spp. D. Yersinia spp.

B In stages I and II of Lyme disease, EIA testing is performed on serum for antibodies. PCR testing and culture of Borrelia spp. are performed on a skin biopsy. In stage III, synovial fluid, skin biopsy, and CSF are tested for Borrelia spp. by PCR.

Following a hike in the woods, a young male noted a tick on his ankle. He removed the tick, but 2 weeks later noticed a circular, bull's eye rash at the site of the bite. Which specimen(s) should be obtained to establish a diagnosis of Lyme borreliosis? A. Lymph node biopsy, skin scraping B. Blood, CSF, and skin biopsy C. Hair, fingernails D. Saliva, sputum

D Typically, the IMViC reactions for the organisms listed are: E. coli (++00) S. typhi (0+00) Y. enterocolitica (V+00) P. vulgaris (++00)

Four blood cultures were taken over a 24-hour period from a 20-year-old woman with severe diarrhea. The cultures grew motile (room temperature), gram-negative rods. A urine specimen obtained by catheterization also showed gram-negative rods, 100,000 col/mL. Given the following results, which is the most likely organism? TSI = A/A gas Indole = + VP = Neg MR = + H2S = Neg Citrate = Neg Urease = Neg Lysine decarboxylase = + Phenylalanine deaminase = Neg A. Proteus vulgaris B. Salmonella typhi C. Yersinia enterocolitica D. E. coli

A Staphylococci are susceptible to furazolidone, giving zones of inhibition that are 15 mm or greater. Micrococcus spp. are resistant to furazolidone, giving zones of 6-9 mm. The test is performed as a disk susceptibility procedure using a blood agar plate.

Furazolidone (Furoxone) susceptibility is a test used to differentiate: A. Staphylococcus spp. from Micrococcus spp. B. Streptococcus spp. from Staphylococcus spp. C. Staphylococcus spp. from Pseudomonas spp. D. Streptococcus spp. from Micrococcus spp.

A A. israelii is part of the normal flora of the mouth and tonsils but may cause upper or lower respiratory tract infections. The sulfur granules are granular microcolonies with a purulent exudate. Like Nocardia, Actinomyces produces unbranched mycelia and is sometimes (erroneously) considered a fungus. It has also been implicated in pelvic infection associated with intrauterine contraceptive devices (IUDs).

Gram stain of a smear taken from the periodontal pockets of a 30-year-old man with poor dental hygiene showed sulfur granules containing gram-positive rods (short diphtheroids and some unbranched filaments). Colonies on blood agar resembled "molar teeth" in formation. The most likely organism is: A. Actinomyces israelii B. Propionibacterium acnes C. Staphylococcus intermedius D. Peptostreptococcus anaerobius

D All of the listed Neisseria spp. grow on MTM and are oxidase positive. N. lactamica is a nonpathogenic component of normal throat flora resembling N. meningitidis but it grows well on selective MTM agar. Presumptive identification of N. meningitidis or N. gonorrhoeae is stated only if the source of the specimen (i.e., urogenital or CSF) is given. The identification must be confirmed by further testing such as carbohydrate utilization tests, DNA tests, or rapid latex slide agglutination tests.

Gram-negative diplococci recovered from an MTM plate and giving a positive oxidase test can be presumptively identified as: A. Neisseria gonorrhoeae B. Neisseria meningitidis C. Neisseria lactamica D. All of these options

B Spores are generally not demonstrated from clinical specimens containing C. perfringens, which is the only species producing a double zone of hemolysis. The reactions in the chart above distinguish the four species listed.

Gram-positive bacilli recovered from two blood cultures from a 60-year-old diabetic patient gave the following results: Spores seen = Neg Hemolysis = + (double zone) Motility = Neg Lecithinase = + Volatile acids by GLC (PYG) = acetic acid (A) and butyric acid (B) What is the most likely identification? A. Clostridium tetani B. Clostridium perfringens C. Clostridium novyi (B) D. Clostridium sporogenes

A M. bovis and M. tuberculosis are very similar biochemically, and some strains of M. bovis also accumulate niacin. The T2H test differentiates M. tuberculosis from M. bovis. M. tuberculosis is not inhibited by T2H.

Growth inhibition by thiophene-2-carboxylic hydrazide (T2H) is used to differentiate M. tuberculosis from which other Mycobacterium specie? A. M. bovis B. M. avium-intracellulare complex C. M. kansasii D. M. marinum

A H. influenzae and subspecies H. aegyptius are both glucose, urease, oxidase, and catalase positive. H. influenzae (biotype II) is positive for both indole and xylose, whereas H. aegyptius is negative for both tests. Biotype II encompasses 40%-70% of H. influenzae strains recovered from clinical specimens. H. influenzae subspecies aegyptius is responsible for epidemics of conjunctivitis in children.

Haemophilus influenzae causes ocular infections (pinkeye) and requires X and V factors in the primary medium for growth. The subspecies Haemophilus influenza (biogroup) aegyptius can further be identified and differentiated by which two tests? A. Indole and xylose B. Glucose and urease C. Oxidase and catalase D. ALA test and oxidase

C The V factor, NAD, must first be released from RBCs before it can be assimilated by Haemophilus spp. Chocolate agar is made by heating blood agar in order to lyse RBCs. The released NAD is directly available to those Haemophilus species requiring it. Chocolate agar also contains the X factor (hemin). All Haemophilus except H. ducreyi and H. aphrophilus require V factor, while X factor is required by H. influenzae, H. haemolyticus, and H. ducreyi.

Haemophilus species that require the V factor (NAD) are easily recovered on which primary agar plate? A. Blood agar made with sheep red cells B. Blood agar made with horse red cells C. Chocolate agar D. Xylose agar

D Standard therapy using INH and rifampin for classic, uncomplicated pulmonary tuberculosis is 9 months. The patient may not respond to therapy, even when the organism is susceptible to the antibiotics in vitro; therefore, cultures must be kept for up to 1 year in order to facilitate testing of additional antibiotics should the infection become refractory to therapy.

How long should Mycobacterium tuberculosis-positive cultures be kept by the laboratory after identification and antibiotic susceptibility testing have been performed? A. 1-2 months B. 2-4 months C. 5-6 months D. 6-12 months

D The culture and Gram stain of the puncture wound site usually does not produce any evidence of C. tetani. The diagnosis is usually based upon clinical findings, which are characterized by spastic muscle contractions, lockjaw, and backward arching of the back caused by muscle contraction.

Identification of Clostridium tetani is based upon: A. Gram stain of the wound site B. Anaerobic culture of the wound site C. Blood culture results D. Clinical findings

A Chlamydophila psittaci is the new taxonomic classification for Chlamydia psittaci. This bacterium is found naturally in psittacine birds and other avian species. Human infection, psittacosis, is a result of contact with pet birds, or from occupational contact in poultry farming or processing. Inhalation of the organisms from aerosols, fecal material, or feather dust causes a respiratory infection and the specimen of choice is a throat swab or sputum.

Identify the following bacterium and specimen pairing that is mismatched (specimen not appropriate for isolation). A. Chlamydia (Chlamydophila) psittaci: fecal swab B. Chlamydia trachomatis: first voided urine C. Chlamydia trachomatis: endocervical swab D. Chlamydia pneumonia: throat swab or sputum

18. A B. abortus can be differentiated from B. melitensis by the following reactions: CO2 Require- Basic Thionin H2S Urease ment Fuchsin (20 mg) B. melitensis Neg V Neg + + B. abortus + + +/V + Neg

In addition to CO2 requirements and biochemical characteristics, Brucella melitensis and Brucella abortus are differentiated by growth on media containing which two dyes? A. Basic fuchsin and thionin B. Methylene blue and crystal violet C. Carbol fuchsin and iodine D. Safranin and methylene blue

B Approximately 80% of group B streptococci are capable of growing in 6.5% salt broth; however, they do not hydrolyze esculin or grow in media containing 4% bile salts.

In addition to Enterococcus faecalis, which other streptococci will grow in 6.5% salt broth? A. Group A streptococci B. Group B streptococci C. Streptococcus pneumoniae D. Group D streptococci (nonenterococci)

B The two genera, Acinetobacter and Alcaligenes, are very similar. Both use oxidation for the metabolism of carbohydrate, with some strains being nonsaccharolytic. Both grow well on MacConkey agar. However, Acinetobacter is nonmotile and oxidase negative. Alcaligenes is motile by peritrichous flagella and oxidase positive.

In addition to motility, which test best differentiates Acinetobacter spp. and Alcaligenes spp.? A. TSI B. Oxidase C. Catalase D. Flagellar stain

D The Kaufmann-White schema groups the salmonellae on the basis of the somatic O (heat-stable) antigens and subdivides them into serotypes based on their flagellar H (heat-labile) antigens. The Vi (or K) antigen is a capsular polysaccharide that may be removed by heating. There are over 2,200 serotypes of Salmonella.

In the Kauffmann-White schema, the combined antigens used for serological identification of the Salmonella spp. are: A. O antigens B. H antigens C. Vi and H antigens D. O, Vi, and H antigens

C A positive PPD test indicates a person who is latently infected with M. tuberculosis. Such persons are asymptomatic and not infectious, but have a 10% risk of developing tuberculosis during their lifetime.

Individuals showing a positive purified protein derivative (PPD) skin test for M. tuberculosis are usually: A. Infective B. Symptomatic of pulmonary disease C. Latently infected D. Falsely positive

A Both Kingella kingae and E. corrodens are gram- negative rods that are oxidase positive and catalase negative. Both grow well on blood and chocolate agars and cause pitting of the media, and neither grows on MacConkey or XLD agar. However, K. kingae strains produce a narrow zone of β-hemolysis on sheep blood agar similar to that of group B streptococci.

Kingella kingae can best be differentiated from Eikenella corrodens using which medium? A. Sheep blood agar B. Chocolate agar C. MacConkey agar D. XLD agar

B Kingella spp. are gram-negative coccobacilli or plump-looking rods. They are part of the normal flora of the upper respiratory and urogenital tracts of humans. Infection is seen primarily in patients having underlying heart disease, poor oral hygiene, or iatrogenic mucosal ulcerations (e.g., radiation therapy), in whom the organism is recovered from blood cultures.

Kingella kingae is usually associated with which type of infection? A. Middle ear B. Endocarditis C. Meningitis D. Urogenital

C Both KIA and TSI contain 10-fold more lactose than glucose, peptone, and phenol red to detect acid production (turns yellow) and sodium thiosulfate and ferrous ammonium sulfate to detect H2S production. However, TSI contains sucrose and KIA does not. Organisms fermenting either sucrose or lactose will turn the slant of the agar tube yellow. Therefore, some organisms (e.g., many species of Cedecea, Citrobacter, Edwardsiella, and Serratia) will produce a yellow slant on TSI but a red slant on KIA.

Kligler iron agar (KIA) differs from triple-sugar iron agar (TSI) in the: A. Ratio of lactose to glucose B. Ability to detect H2S production C. Use of sucrose in the medium D. Color reaction denoting production of acid

B The only spore former listed is the Bacillus spp., which grow as large, spreading colonies on blood agar plates. Pseudomonas spp. are gram-negative rods; Corynebacterium spp. appear as small, very dry colonies on BAP; Listeria spp. appear as very small β-hemolytic colonies on BAP, resembling Streptococcus species.

Large gram-positive spore-forming rods growing on blood agar as large, raised, β-hemolytic colonies that spread and appear as frosted green-gray glass are most likely: A. Pseudomonas spp. B. Bacillus spp. C. Corynebacterium spp. D. Listeria spp.

C LIA is used as an aid for the identification of Salmonella species. It contains phenylalanine, lysine, glucose, thiosulfate, ferric ammonium citrate, and bromcresol purple. Salmonella produce H2S from thiosulfate. This reduces ferric ammonium citrate, forming ferrous sulfate and causing the butt to blacken. Salmonella also decarboxylate lysine to produce alkaline amines, giving the slant its purple color and differentiating it from Citrobacter spp., which are lysine decarboxylase negative.

Lysine iron agar (LIA) showing a purple slant and a blackened butt indicates: A. E. coli B. Citrobacter spp. C. Salmonella spp. D. Proteus spp.

C Lysostaphin is an endopeptidase that cleaves the glycine-rich pentapeptide crossbridges in the staphylococcal cell wall peptidoglycan. The susceptibility of the staphylococci to lysostaphin is used to differentiate them from the micrococci. Staphylococci are susceptible and show a 10-16 mm zone of inhibition, while micrococci are not inhibited.

Lysostaphin is used to differentiate Staphylococcus from which other genus? A. Streptococcus B. Stomatococcus C. Micrococcus D. Planococcus

A Both N. gonorrhoeae and N. meningitidis grow selectively on MTM owing to the addition of vancomycin and colistin, which inhibit gram-positive and gram-negative bacteria, respectively. Trimethoprim is added to inhibit swarming of Proteusspp. because a rectal swab may be used for culture. Nystatin and amphotericin B are used to prevent growth of yeasts and molds from vaginal specimens.

MTM medium is used primarily for the selective recovery of which organism from genital specimens? A. Neisseria gonorrhoeae B. Neisseria lactamica C. Neisseria sicca D. Neisseria flavescens

A E. faecalis is resistant to penicillin and ampicillin as well as some of the aminoglycoside antibiotics. Pneumococci, group B streptococci, and S. bovis are PYR negative.

Many α-hemolytic streptococci recovered from a wound were found to be penicillin resistant. Given the following results, what is the most likely identification? Bile esculin = + PYR = + 6.5% salt = + Hippurate Bile solubility= Neg hydrolysis = + SXT = Resistant A. Enterococcus faecalis B. Streptococcus pneumoniae C. Streptococcus bovis D. Group B streptococci

B Anaerobic bacteria can be identified by analysis of metabolic products using gas-liquid chromatography. Results are evaluated along with Gram staining characteristics, spore formation, and cellular morphology in order to make the identification.

Methods other than packaged microsystems used to identify anaerobes include: A. Antimicrobial susceptibility testing B. Gas-liquid chromatography (GLC) C. Special staining D. Enzyme immunoassay

A Both micrococci and staphylococci are catalase-positive and gram-positive cocci. On direct smears, they both appear as pairs, short chains (resembling Streptococcus spp.), or clusters. However, the micrococci fail to produce acid from glucose under anaerobic conditions. The OF tube reactions are:

Micrococcus and Staphylococcus species are differentiated by which test(s)? A. Fermentation of glucose (OF tube) B. Catalase test C. Gram stain D. All of these options

A M. xenopi causes a pulmonary infection resembling M. tuberculosis and is frequently isolated from patients with an underlying disease such as alcoholism, AIDS, diabetes, or malignancy. It is often recovered from hot water taps and contaminated water systems and is a possible source of nosocomial infection. The other three species cause skin infections and grow on artificial media at a much lower temperature than M. xenopi (below 32°C).

Mycobacteria isolated from the hot water system of a hospital grew at 42°C. Colonies on Löwenstein-Jensen medium were not pigmented after exposure to light and were negative for niacin accumulation and nitrate reduction. The most likely identification is: A. Mycobacterium xenopi B. Mycobacterium marinum C. Mycobacterium ulcerans D. Mycobacterium haemophilum

A The carbolfuchsin (fuchsin with phenol) stains the mycobacteria red and does not decolorize after the acid-alcohol is added. The background and any other bacterial elements will decolorize and are counterstained blue by the methylene blue. A fluorescent staining procedure may be used as an alternative to acid-fast staining. Auramine fluorochrome produces bright yellow fluorescent mycobacteria and auramine-rhodamine causes an orange-red (gold) fluorescence against a dark background. A fluorescent microscope must be used, but with this method the smear can be scanned with a 25× objective instead of the 100× objective, permitting more rapid identification of mycobacteria.

Mycobacteria stained by the Ziehl-Neelsen or Kinyoun methods with methylene blue counterstain are seen microscopically as: A. Bright red rods against a blue background B. Bright yellow rods against a yellow background C. Orange-red rods against a black background D. Bright blue rods against a pink background

B Moraxella spp. are oxidase and catalase positive, as are the gonococci. Neisseria spp. and M. catarrhalis will keep their typical coccal morphology after overnight incubation on blood agar with a 10-unit penicillin disk (CO2 incubation). Other Moraxella species form long filaments or long spindle-shaped cells when grown near a 10-unit penicillin disk.

Nonpathogenic Moraxella spp. capable of growing on selective media for Neisseria can be differentiated from Neisseria spp. by which test? A. Catalase test B. 10-unit penicillin disk C. Oxidase test D. Superoxol test

D The staphylococcal streak, across the NVS inoculum, provides the nutrients needed. Very small colonies of NVS can be seen growing adjacent to the staphylococcal streak on the blood agar plate in a manner similar to the satellite phenomenon of Haemophilus spp. around S. aureus.

Nutritionally variant streptococci (NVS) require specific thiol compounds, cysteine, or the active form of vitamin B6. Which of the following tests supplies these requirements? A. CAMP test B. Bacitracin susceptibility test C. Bile solubility test D. Staphylococcal cross-streak test

B A common cause of respiratory tract illness, M. pneumoniae, generally causes a self-limited infection (3-10 days) and usually does not require antibiotic therapy. M. pneumoniae can be cultured from the upper and lower respiratory tracts onto specially enriched (diphasic) media, but is most frequently diagnosed by the change in antibody titer from acute to convalescent serum using enzyme immunoassay or other serological methods.

Primary atypical pneumonia is caused by: A. Streptococcus pneumoniae B. Mycoplasma pneumoniae C. Klebsiella pneumoniae D. Mycobacterium tuberculosis

D The anaerobic bacteria are subdivided according to their requirement for O2. Obligate anaerobes are killed by exposure to atmospheric O2 for 10 min or longer. Facultative anaerobes grow under aerobic or anaerobic conditions. Microaerophilic organisms do not grow in an aerobic incubator on solid media and only minimally under anaerobic conditions. However, they will grow in minimal oxygen (5% O2). Superoxide dismutase (SOD) is produced by many anaerobes, which catalyzes the conversion of superoxide radicals to less toxic H2O2 and molecular O2.

Obligate anaerobes, facultative anaerobes, and microaerophiles are terms referring to bacteria that require: A. Increased nitrogen B. Decreased CO2 C. Increased O2 D. Decreased O2

B The Bacteroides group grows well in 20% bile and is resistant to penicillin 2-unit disks with the exception of B. ureolyticus. Most Prevotella are also resistant to penicillin 2-unit disks, but most Fusobacterium and Porphyromonas are sensitive.

Obligate anaerobic gram-negative bacilli that do not form spores grow well in 20% bile and are resistant to penicillin 2-unit disks are most likely: A. Porphyromonas spp. B. Bacteroides spp. C. Fusobacterium spp. D. Prevotella spp.

B (Anaerobic culture media can be prereduced before sterilization by boiling, saturation with oxygen-free gas, and addition of cysteine or other thiol compounds. The final oxidation reduction potential (Eh) of the medium should be approximately -150 mV to minimize the effects of exposure of organisms to oxygen during inoculation.)

Prereduced and vitamin K1-supplemented blood agar plates are recommended isolation media for: A. Mycobacterium marinum and Mycobacterium avium intracellulare B. Bacteroides, Peptostreptococcus, and Clostridium spp. C. Proteus spp. D. Enterococcus spp.

D M. tuberculosis is a slow grower with a prolonged culture time of 12-25 days and requires 3-6 weeks for definitive identification and antibiotic susceptibility testing. The acid-fast smear remains the number one rapid test for the detection of mycobacterial infection. A positive smear has a predictive value of 96% when all laboratory and clinical findings are considered. GLC is used to evaluate cell wall lipid patterns for identification. DNA probes are available for rapid identification of M. tuberculosis, M. bovis, M. avium-intracellulare complex, and M. gordonae. A commonly used PCR method for mycobacterial detection involves amplification of a species-specific region of DNA with a labeled (biotinylated) oligonucleotide primer. The PCR product is detected by denaturation and hybridization to a capture probe. After washing to remove unbound DNA, strepavidin conjugated to an enzyme is added. After washing to remov

Rapid methods for identifying classic infection with M. tuberculosis include: A. Gas-liquid chromatography B. Nucleic acid probes C. Acid-fast smears D. All of these options

A Oxacillin is the drug used to screen staphylococci for resistance to antibiotics having the β-lactam ring. Included in this group are penicillin, cephalosporin, monobactam, and carbapenem. MRSA defines strains of staph that are resistant to all of these antibiotic groups. MRSA strains are treated with vancomycin or oxazolidinone.

Resistance to which drug categorizes a strain of Staphylococcus aureus as methicillin-resistant Staphylococcus aureus (MRSA)? A. Oxacillin B. Colistin C. Trimethoprim-sulfamethoxazole D. Tetracycline

B Serological tests of the patient's serum for evidence of syphilis are routinely performed, but culturing is not because research animals must be used for inoculation of the suspected spirochete. T. pallidum does not stain by either the Gram or acid-fast technique. Darkfield microscopy for direct visualization or indirect immunofluorescence using fluorescein-conjugated antihuman globulin (the fluorescent treponemal antibody-absorption test, FTA-ABS) may be used to identify syphilis. Newer tests for specific antibodies to T. pallidum are available in a wide range of immunoassay formats including chemiluminescence and point-of-care immunochromatography. T. pallidum infection can be diagnosed by PCR. Because the bacterium is fastidious, blood samples should be collected in EDTA, CSF should be frozen and sent on dry ice, and samples analyzed as soon as possible.

Routine laboratory testing for Treponema pallidum involves: A. Culturing B. Serological analysis C. Acid-fast staining D. Gram staining

A Optochin at a concentration of 5 μg/mL or less inhibits the growth of S. pneumoniae but not viridans streptococci. However, Optochin at a concentration in excess of 5 μg/mL inhibits viridans streptococci as well. A zone of inhibition of 14 mm or more around the 6-mm disk is considered a presumptive identification of S. pneumoniae. A questionable zone size should be confirmed by performing a bile solubility test.

S. pneumoniae and the viridans streptococci can be differentiated by which test? A. Optochin disk test, 5 μg/mL or less B. Bacitracin disk test, 0.04 unit C. CAMP test D. Bile esculin test

C (E. coli 0157:H7 ferments lactose, and therefore, appears as dark pink colonies on MacConkey agar. To differentiate E. coli 0157:H7 from normal fecal flora, MacConkey agar with sorbitol is used. E. coli 0157:H7 does not ferment sorbitol, and usually are colorless colonies.)

SITUATION: A group of elementary students became ill after eating undercooked ground beef prepared in the school cafeteria. The suspected pathogen, E. coli serotype 0157:H7, is usually recovered using which of the following media? A. XLD agar B. MacConkey agar C. MacConkey agar with sorbitol D. Hektoen agar

B (TCBS agar is used to grow Vibrio cholerae, which appear as yellow colonies as a result of the use of both citrate and sucrose. APW is used as an enrichment broth and should be subcultured to TCBS agar for further evaluation of Vibrio colonies.)

Select the media of choice for recovery of Vibrio cholerae from a stool specimen. A. MacConkey agar and thioglycollate media B. Thiosulfate-citrate-bile-sucrose (TCBS) agar and alkaline peptone water (APW) broth C. Blood agar and selenite-F (SEL) broth D. CNA agar

B (CIN agar inhibits the growth of many other organisms from the family Enterobacteriaceae. Yersinia spp. are also recovered from MacConkey and Salmonella-Shigella agars.)

Select the medium best suited for the recovery of Yersinia enterocolitica from a patient with gastroenteritis. A. Hektoen agar B. Cefsulodin-Irgasan-Novobiocin (CIN) agar C. Blood agar D. Eosinmethylene blue agar

C (Anaerobic specimens are easily contaminated with organisms present on the skin or mucosal surfaces when a swab is used. Needle aspiration of an abscess following surface decontamination provides the least exposure to ambient oxygen.)

Select the method of choice for recovery of anaerobic bacteria from a deep abscess. A. Cotton fiber swab of the abscess area B. Skin snip of the surface tissue C. Needle aspirate after surface decontamination D. Swab of the scalpel used for débridement

B S. epidermidis produces an extracellular slime that enhances the adhesion of these organisms to indwelling plastic catheters. The slime production is considered a virulence factor and is associated with infections from prostheses.

Slime production is associated with which Staphylococcus species? A. S. aureus B. S. epidermidis C. S. intermedius D. S. saprophyticus

B Pasteurella multocida (P. canis) is part of the normal mouth flora of cats and dogs and is frequently recovered from wounds inflicted by them. It produces large amounts of indole and therefore an odor resembling colonies of E. coli. Pseudomonas spp. are also catalase and oxidase positive but can be ruled out because they grow on MacConkey agar and do not produce indole.

Smooth gray colonies showing no hemolysis are recovered from an infected cat scratch on blood and chocolate agar but fail to grow on MacConkey agar. The organisms are gram-negative pleomorphic rods that are both catalase and oxidase positive and strongly indole positive. The most likely organism is: A. Capnocytophaga spp. B. Pasteurella spp. C. Proteus spp. D. Pseudomonas spp.

D (Media for transporting specimens for virus culture include Hanks balanced salt solution with bovine albumin, Stuart transport media, and Leibovitz-Emory media. Media used for transporting specimens for viral culture are similar to those for bacteria with the addition of a nutrient such as fetal calf serum or albumin and antibiotics. Specimens should be refrigerated after being placed in the transport media until the culture media can be inoculated.)

Specimens for virus culture should be transported in media containing: A. Antibiotics and 5% sheep blood B. Saline and 5% sheep blood C. 22% bovine albumin D. Antibiotics and nutrient

A Borrelia spp. are often seen on Wright's-stained smears of peripheral blood as helical bacteria with 3-10 loose coils. They are gram negative but stain well with Giemsa's stain.

Spirochetes often detected in the hematology laboratory, even before the physician suspects the infection, are: A. Borrelia spp. B. Treponema spp. C. Campylobacter spp. D. Leptospira spp.

A M. catarrhalis is part of the normal upper respiratory flora but is implicated in lower respiratory infections, including pneumonia. It produces stunted growth on MTM and is DNase positive, characteristics that differentiate it from the other saprophytic Neisseria species.

Sputum from a patient with pneumonia produced many colonies of gram-negative diplococci on a chocolate plate that were also present in fewer numbers on MTM after 48 hours. Given the following results, what is the most likely identification? Catalase = + Oxidase = + DNase = + Tributyrin hydrolysis = + Glucose = Neg Sucrose = Neg Lactose = Neg Maltose = Neg Fructose = Neg A. Moraxella catarrhalis B. Neisseria flavescens C. Neisseria sicca D. Neisseria elongata

D Vancomycin, along with rifampin, is used for strains of S. aureus that are resistant to the β-lactams. MRSA strains pose problems when reading the zone sizes for these strains. Their heteroresistance results in a film of growth consisting of very small colonies formed within the defined inhibition zone surrounding the antibiotic disk. Initially, this appears as a mixed culture or contaminant.

Staphylococcus aureus recovered from a wound culture gave the following antibiotic sensitivity pattern by the standardized Kirby-Bauer method (S = sensitive; R = resistant): Penicillin = R Ampicillin = S Cephalothin = R Cefoxitin = R Vancomycin = S Methicillin = R Which is the drug of choice for treating this infection? A. Penicillin B. Ampicillin C. Cephalothin D. Vancomycin

D S. epidermidis represents 50%-80% of all coagulase-negative Staphylococcus spp. recovered from numerous clinical specimens. It is of special concern in nosocomial infections because of its high resistance to antibiotics.

Staphylococcus epidermidis (coagulase negative) is recovered from which of the following sources? A. Prosthetic heart valves B. Intravenous catheters C. Urinary tract D. All of these options

B S. saprophyticus is coagulase negative and resistant to 5 μg of novobiocin. Using the standardized Kirby-Bauer sensitivity procedure, a 6-12 mm zone of growth inhibition is considered resistant. Susceptible strains measure 16-27 mm (inhibition) zones.

Staphylococcus saprophyticus is best differentiated from Staphylococcus epidermidis by resistance to: A. 5 μg of lysostaphin B. 5 μg of novobiocin C. 10 units of penicillin D. 0.04 unit of bacitracin

B Streptococcus species are facultative anaerobes that grow aerobically as well, and are oxidase and catalase negative. In order to demonstrate streptolysin O on blood agar, it is best to stab the agar to create anaerobiosis because streptolysin O is oxygen labile.

Streptococcus species exhibit which of the following properties? A. Aerobic, oxidase positive, and catalase positive B. Facultative anaerobe, oxidase negative, catalase negative C. Facultative anaerobe, β-hemolytic, catalase positive D. May be α-, β-, or γ-hemolytic, catalase positive

B Blood cultures growing small gram-negative rods shoud alert the microbiologist to the possiblility of infection with one of the five HACEK organisms. Although responsible for less than 5% of bacterial endocarditis overall, greater than half of endocarditis cases caused by gram-negative rods result from one of them.

The HACEK group of organisms (Haemophilus aphrophilus, Actinobacillus actinomycetemcomitans, Cardiobacterium hominis, Eikenella corrodens, and Kingella spp.) are all known for which type of infection? A. Urinary tract B. Endocarditis C. Pharyngitis D. Tonsillitis

A The PYR hydrolysis test is highly specific for group A streptococci and group D enterococci. The test detects the pyrrolidonylarylamidase enzyme, which hydrolyzes PYR.

The L-pyrrolidonyl-β-napthylamide (PYR) hydrolysis test is a presumptive test for which streptococci? A. Group A and D (enterococcus) streptococci B. Group A and B β-hemolytic streptococci C. Nongroup A or B β-hemolytic streptococci D. Streptococcus pneumoniae and group D streptococci (nonenterococcus)

B Strains of E. coli that produce one or both of the Shiga-like toxins (SLT I and SLT II) can cause bloody diarrhea (hemorrhagic colitis). In the United States, E. coli strain O157:H7 is the serotype most often associated with hemorrhagic colitis.

The Shiga-like toxin (verotoxin) is produced mainly by which Enterobacteriaceae? A. Klebsiella pneumoniae B. E. coli C. Salmonella typhimurium D. Enterobacter cloacae

A N. gonorrhoeae colonies recovered from selective MTM media give an immediate positive reaction (bubbling) when 30% H2O2 is added. The catalase test uses 3% H2O2. This is a presumptive test for N. gonorrhoeae; N. meningitidis and N. lactamica give a weak or delayed bubbling reaction. M. catarrhalis is catalase positive, Superoxol negative, and has a variable growth pattern on MTM.

The Superoxol test is used as a rapid presumptive test for: A. Neisseria gonorrhoeae B. Neisseria meningitidis C. Neisseria lactamica D. Moraxella (Branhamella) catarrhalis

A Contamination by fungi and other bacteria contributes to lower yields of mycobacteria in a 24-hour sample. The first morning specimen collected by expectoration or nebulization produces the highest concentration of mycobacteria in sputum.

The best specimen for recovery of the mycobacteria from a sputum sample is: A. First morning specimen B. 10-hour evening specimen C. 12-hour pooled specimen D. 24-hour pooled specimen

B The bile solubility test can be performed directly by dropping 2% sodium deoxycholate onto a few well-isolated colonies of S. pneumoniae. The bile salts speed up the autolysis observed in pneumococcal cultures. The colonies lyse and disappear when incubated at 35°C for 30 min, leaving a partially hemolyzed area on the plate. The same phenomenon can be seen using a broth culture; addition of 10% deoxycholate to broth containing S. pneumoniae results in visible clearing of the suspension after incubation at 35°C for 3 hours.

The bile solubility test causes the lysis of: A. Streptococcus bovis colonies on a blood agar plate B. Streptococcus pneumoniae colonies on a blood agar plate C. Group A streptococci in broth culture D. Group B streptococci in broth culture

D One milliliter of an equal mixture of 30% H2O2 (Superoxol) and Tween 80 (a strong detergent) is added to a 2-week-old subculture on Löwenstein-Jensen medium and placed upright for 5 minutes. Catalase activity is determined semiquantitatively by measuring the height of the column of bubbles produced above the culture surface.

The catalase test for mycobacteria differs from that used for other types of bacteria by using: A. 1% H2O2 and 10% Tween 80 B. 3% H2O2 and phosphate buffer, pH 6.8 C. 10% H2O2 and 0.85% saline D. 30% H2O2 and 10% Tween 80

B Foodborne botulism in adults and children is caused by ingestion of the preformed toxin (botulinum toxins A, B, E, and F) in food. The neurotoxins of C. botulinum are protoplastic proteins made during the growing phase and released during lysis of the organisms. Confirmation of botulism is made by demonstration of the toxin in serum, gastric, or stool specimens.

The classic form of foodborne botulism is characterized by the ingestion of: A. Spores in food B. Preformed toxin in food C. Toxin H D. All of these options

A Serological analysis using immunofluorescence or an enzyme immunoassay is the method of choice for diagnosis of Lyme disease. Titers of IgM remain high throughout the infection. B. burgdorferi can be cultured directly from lesions, and darkfield microscopy can be used for detection of spirochetes in blood cultures after 2-3 weeks of incubation at 34°C-37°C.

The diagnostic method most commonly used for the identification of Lyme disease is: A. Serology B. Culture C. Gram stain D. Acid-fast stain

B Fusobacterium spp. are usually spindle-shaped, slim rods, whereas the other genera are small rods (variable length for Bacteroides spp. and tiny coccoid rods for Prevotella and Porphyromonas spp.). Fusobacterium spp. and Porphyromonas spp. are susceptible to penicillin 2-unit disks, while most Bacteroides spp. and Prevotella spp. are resistant.

The following characteristics of an obligate anaerobic gram-negative bacilli best describe which of the listed genera? Gram stain: long, slender rods with pointed ends Colonial appearance: dry bread crumbs or "fried-egg" appearance Penicillin 2-unit disk test: Susceptible A. Bacteroides spp. B. Fusobacterium spp. C. Prevotella spp. D. Porphyromonas spp.

C S. aureus can produce fibrinolysins that dissolve the clot formed by the coagulase enzyme. The tube method calls for an incubation of 4 hours at 35°C-37°C and 18-24 hours at room temperature. Both must be negative to interpret the result as coagulase negative. This organism is coagulase positive and, therefore, identified as S. aureus.

The following results were observed by using a tube coagulase test: Coagulase at 4 hours = + Coagulase at 18 hours = Neg DNase = + Novobiocin = Sensitive (16-mm zone) Hemolysis on blood Mannitol salt plate = + agar = β (acid production) What is the most probable identification? A. Staphylococcus saprophyticus B. Staphylococcus epidermidis C. Staphylococcus aureus D. Staphylococcus hominis

A P. acnes is a nonspore former and is described as a diphtheroid-shaped rod. It is part of the normal skin, nasopharynx, genitourinary, and gastrointestinal tract flora but is implicated as an occasional cause of endocarditis.

The gram-positive non-spore-forming anaerobic rods most frequently recovered from blood cultures as a contaminant are: A. Propionibacterium acnes B. Clostridium perfringens C. Staphylococcus intermedius D. Veillonella parvula

B The majority of H. influenzae infections occur in children under 5 years old and are caused by capsular serotype b, one of six serotypes designated a through f. This strain appears to contain a virulence factor that makes it resistant to phagocytosis and intracellular killing by neutrophils. Serotyping of Haemophilus is performed by mixing colonies with agglutinating antibodies available as commercial agglutination kits.

The majority of Haemophilus influenzae infections are caused by which of the following capsular serotypes? A. a B. b C. c D. d

B U. urealyticum is the only human mycoplasma that hydrolyzes urea. The manganous chloride-urea test utilizes manganous chloride (MnCl2) in the presence of urea. Urease produced by the organism hydrolyzes the urea to ammonia. This reacts with MnCl2 forming manganese oxide, which is insoluble and forms a dark brown precipitate around the colonies. The reaction is observed under a dissecting microscope and is a rapid test for the identification of U. urealyticum.

The manganous chloride-urea test is used for the identification of which organism? A. Mycoplasma pneumoniae B. Ureaplasma urealyticum C. Bacillus cereus D. Borrelia burgdorferi

D A precipitin reaction seen microscopically with methylene blue stain (microprecipitin reaction) occurs between the carbohydrate of the capsule of S. pneumoniae and anticapsular antibody. The antibody may be type specific or polyvalent. Binding of antibodies to the bacteria causes the capsule to swell, identifying the organisms as S. pneumoniae.

The quellung test is used to identify which Streptococcus species? A. S. pyogenes B. S. agalactiae C. S. sanguis D. S. pneumoniae

D Enterococcus faecalis will grow in 6.5% salt and the nonenterococci (S. bovis and S. equinus) will not. This test distinguishes the enterococci group from S. bovis and S. equinus (nonenterococci group). Both groups grow on bile esculin agar.

The salt tolerance test (6.5% salt broth) is used to presumptively identify: A. Streptococcus pneumoniae B. Streptococcus bovis C. Streptococcus equinus D. Enterococcus faecalis

B The catalase test (utilizing a 3% hydrogen peroxide [H2O2] solution stored in a brown bottle under refrigeration) is positive for the four genera belonging to the Micrococcaceae family: Planococcus, Micrococcus, Stomatococcus, and Staphylococcus. Members of the Streptococcaceae family are negative. Planococcusspp. are associated with marine life and not human infections. Stomatococcus spp. are implicated in endocarditis following cardiac catheterization; they are weakly catalase positive and produce white or transparent sticky colonies on agar, which help to differentiate them from Staphylococcus.

The test used most often to separate the Micrococcaceae family from the Streptococcaceae family is: A. Bacitracin B. Catalase C. Hemolysis pattern D. All of these options

A Both pathogenic and saprophytic mycobacteria may produce urease, and urease production is used to differentiate several mycobacteria species. Biochemically, M. scrofulaceum is identical to M. gordonae, except for the urease reaction for which M. scrofulaceum is positive and M. gordonae is negative. Urease reactions for the other pathogenic mycobacteria are: M. tuberculosis = + M. kansasii = + M. bovis = + M. avium-intracellulare complex = Neg

The urease test is needed to differentiate Mycobacterium scrofulaceum from which of the following mycobacteria? A. M. gordonae B. M. kansasii C. M. avium-intracellulare complex D. M. bovis

A N. lactamica utilizes lactose by producing the enzyme β-galactosidase. All other Neisseria spp. that grow on MTM media are lactose negative.

The β-galactosidase test aids in the identification of which Neisseria species? A. N. lactamica B. N. meningitidis C. N. gonorrhoeae D. N. flavescens

A The X factor requirement for growth is the cause of many inaccuracies when identifying Haemophilus spp. requiring this factor. False-negative results have been attributed to the presence of small amounts of hemin in the basal media, or X factor carryover from colonies transferred from primary media containing blood. The δ-ALA test determines the ability of an organism to synthesize protoporphyrin intermediates in the biosynthetic pathway to hemin from the precursor compound δ-aminolevulinic acid. Haemophilus species that need exogenous X factor to grow are unable to synthesize protoporphyrin from δ-ALA and are negative for the δ-ALA test. These include H. influenzae, H. haemolyticus, H. aegyptius, and H. ducreyi.

The δ-ALA test (for porphyrins) is a confirmatory procedure for which test used for identification of Haemophilus species? A. X factor requirement B. V factor requirement C. Urease production D. Indole production

C S. aureus is the organism most often recovered from female patients. These strains produce toxic shock syndrome toxin 1 (TSST-1). Toxic shock syndrome is attributed to the use of certain highly absorbent tampons by menstruating females. The toxin is also recovered from sites other than the genital area and produces fever and life-threatening systemic damage as well as shock.

Toxic shock syndrome is attributed to infection with: A. Staphylococcus epidermidis B. Staphylococcus hominis C. Staphylococcus aureus D. Staphylococcus saprophyticus

B β-Hemolytic streptococci are the only streptococci that should be tested. S. pneumoniae, which is α-hemolytic, is susceptible to small concentrations of bacitracin, as are other α-hemolytic streptococci. SXT is inhibitory to most streptococci except Streptococcus pyogenes and Streptococcus agalactiae. For this reason, SXT is used in a commercially available streptococcal selective agar (SSA) as a primary plating agar for the detection of group A streptococci.

Trimethoprim-sulfamethoxazole (SXT) disks are used along with bacitracin disks to differentiate which streptococci? A. α-Hemolytic streptococci B. β-Hemolytic streptococci C. Streptococcus pneumoniae D. Enterococcus faecalis

B Group B streptococci (S. agalactiae) are resistant to both bacitracin and SXT. Unlike group A and group D streptococci, the group B streptococci are negative for PYR. With some exceptions, group B streptococci will grow in 6.5% salt broth.

Two blood cultures on a newborn grew β-hemolytic streptococci with the following reactions: CAMP test = + Hippurate hydrolysis = + Bile solubility = Neg 6.5% salt = + Bacitracin = Resistant Bile esculin = Neg PYR = Neg Trimethoprim-sulfamethoxazole = Resistant Which is the most likely identification? A. Group A streptococci B. Group B streptococci C. Group D streptococci D. Nongroup A, nongroup B, nongroup D streptococci

D S. epidermidis and S. saprophyticus are the two possibilities because they are both catalase positive, coagulase negative, urease positive, and ferment lactose. Novobiocin susceptibility is the test of choice for differentiating these two species. S. epidermidis is sensitive but S. saprophyticus is resistant to 5 μg of novobiocin.

Urine cultured from the catheter of an 18-year-old female patient produced more than 100,000 col/mL on a CNA plate. Colonies were catalase positive, coagulase negative by the latex agglutination slide method as well as the tube coagulase test. The best single test for identification is: A. Lactose fermentation B. Urease C. Catalase D. Novobiocin susceptibility

D Upon subculture from a primary plate, various sizes and appearances of gonococci are the result of multiple nutritional requirements, such as arginine-hypoxanthine-uracil (AHU)-requiring strains. Colony size and coloration (or light reflection) are the basis of Kellogg's scheme (types T1 through T5). Types T1 and T2 have pili on the surface and T3, T4, and T5 do not. Transparent media are not used routinely, but opaque and transparent colonial differences of the gonococci can be seen when using it.

Variation in colony types seen with fresh isolates of Neisseria gonorrhoeae and sometimes with Neisseria meningitidis are the result of: A. Multiple nutritional requirements B. Pili on the cell surface C. Use of a transparent medium D. All of these options

B A strong decontamination solution (6% NaOH or greater) may kill or severely damage the mycobacteria. A 4% NaOH solution is mixed with an equal volume of N-acetyl-L-cysteine (NALC), a digestant or mucolytic agent, to yield a final working concentration of 2% NaOH. The time of exposure of the specimen to the digestion/decontamination solution must be monitored because overtreatment may result in fewer positive cultures.

What concentration of sodium hydroxide (NaOH) is used to prepare a working decontamination solution for the processing of not normally sterile specimens for mycobacteria? A. 1% NaOH B. 4% NaOH C. 8% NaOH D. 12% NaOH

D B. ureolyticus is the only species listed that is susceptible to penicillin and produces urease. The other organisms listed are resistant to penicillin.

Which Bacteroides spp. is noted for "pitting" of the agar and is sensitive to penicillin 2-unit disks? A. B. vulgatus B. B. ovatus C. B. thetaiotaomicron D. B. ureolyticus

D Wounds infected with clostridia are characterized by invasion and liquefactive necrosis of muscle tissue with gas formation. The most frequent isolate is C. perfringens followed by C. novyi and C. septicum.

Which Clostridium species is most often recovered from a wound infection with gas gangrene? A. C. sporogenes B. Clostridium sordellii C. C. novyi D. C. perfringens

B C. difficile is also implicated in hospital acquired diarrhea and colitis. Clinical testing for C. difficile includes culture and cytotoxin testing. Because culture takes 3 days and will detect nontoxigenic strains that do not cause diarrheal disease, immunoassays using antibodies against either the A toxin or both the A and B toxins are most frequently employed. Assays detecting both toxins are only slightly more sensitive, since infections producing only B toxin are infrequent. The cytotoxin assay requires that specimens be shipped to a reference laboratory on dry ice or kept at 4°C-6°C if done in-house.

Which Clostridium spp. causes pseudomembranous colitis or antibiotic-associated colitis? A. C. ramosum B. C. difficile C. C. perfringens D. C. sporogenes

B H. ducreyi requires exogenous X factor and causes genital lesions referred to as "soft chancres." The medium used for recovery is commercial chocolate agar or gonococcus base medium containing 1%-2% hemoglobin, 5% fetal calf serum, and 1% IsoVitaleX enrichment. The plates must be incubated in a 3%-5% CO2 environment for 2-3 days. Most specimens are recovered from heterosexuals, and outbreaks in the United States are traced to female prostitutes.

Which Haemophilus species is difficult to isolate and recover from genital ulcers and swollen lymph nodes? A. H. aphrophilus B. H. ducreyi C. H. haemolyticus D. H. parahaemolyticus

C H. aphrophilus does not require either X or V factor for growth and is differentiated from the other Haemophilus species by its ability to produce acid from lactose and a positive δ-aminolevulinic acid (ALA) test. H. influenzae and H. haemolyticus are incapable of synthesizing protoporphyrin from δ-ALA and are negative for this test.

Which Haemophilus species is generally associated with endocarditis? A. H. influenzae B. H. ducreyi C. H. aphrophilus D. H. haemolyticus

A More than 90% of MAC organisms that are isolated from AIDS patients are M. avium. These are distinguished from M. intraceulluare by NATs.

Which M. avium complex (MAC) organism is the most frequently isolated mycobacterium from AIDS patients? A. M. avium B. M. intracellulare C. M. scrofulaceum D. M. bovis

A S. intermedius infects mammals and certain birds but not usually humans. Cases involving humans result from animal bites and are most often seen in persons who work closely with animals.

Which Staphylococcus species, in addition to S. aureus, also produces coagulase? A. S. intermedius B. S. saprophyticus C. S. hominis D. All of these options

B Both of these bacteria cause diarrhea, grow well on enteric agar, and may be confused with Enterobacteriaceae. Both organisms are positive for oxidase, motility, indole, and lysine decarboxylase. The following reactions are differential:

Which are the best two tests to differentiate A. hydrophilia from P. shigelloides? A. Oxidase and motility B. DNase and VP C. Indole and lysine decarboxylase D. Growth on MacConkey and blood agar

A Campylobacter spp. are best recovered in a micro-aerophilic atmosphere (reduced O2). The use of a CO2 incubator or candle jar is not recommended because the amount of O2 and CO2 do not permit any but the most aerotolerant Campylobacter to survive. Cultures for Campylobacter should be incubated for 48-72 hours before reporting no growth.

Which atmospheric condition is needed to recover Campylobacter spp. from specimens inoculated onto a Campy-selective agar at 35°C-37°C and 42°C? A. 5% O2, 10% CO2, and 85% N2 B. 20% O2, 10% CO2, and 70% N2 C. 20% O2, 20% CO2, and 60% N2 D. 20% O2, 5% CO2, and 75% N2

A Peptone yeast and chopped meat with carbohydrates support the growth of anaerobic bacteria. The end products from the metabolism of the peptone and carbohydrates are volatile fatty acids that help to identify the bacteria. After incubation, the broth is centrifuged, and the supernatant injected into a gas-liquid chromatograph. Peaks for acetic, butyric, or formic acid, for example, can be identified by comparison to the elution time of volatile organic acid standards.

Which broth is used for the cultivation of anaerobic bacteria in order to detect volatile fatty acids as an aid to identification? A. Prereduced peptone-yeast extract-glucose (PYG) B. THIO broth C. Gram-negative (GN) broth D. Selenite (SEL) broth

A Actinobacillus spp. (formerly CDC groups HB-3 and HB-4) share many biochemical characteristics of the Haemophilus spp. Infections most often associated with this gram-negative coccobacillus are subacute bacterial endocarditis and periodontal disease (its main habitat is the mouth). The most common human isolate is Actinobacillus actinomycetemcomitans, which grows slowly on chocolate agar. It is positive for catalase, nitrate reduction, and glucose fermentation. It does not grow on MacConkey agar and is negative for oxidase, urease, indole, X, and V requirements.

Which genus (in which most species are oxidase and catalase positive) of small gram-negative coccobacilli is associated mainly with animals but may cause endocarditis, bacteremia, as well as wound and dental infections in humans? A. Actinobacillus B. Pseudomonas C. Campylobacter D. Vibrio

A Pigmenting Porphyromonas spp. and Prevotella spp. also show hemolysis on sheep blood agar.

Which gram-negative bacilli produce black pigment and brick red fluorescence when exposed to an ultraviolet light source? A. Porphyromonas spp. and Prevotella spp. B. Fusobacterium spp. and Actinomyces spp. C. Bacteroides spp. and Fusobacterium spp. D. All of these options

A Group A β-hemolytic streptococci are the cause of scarlet fever, and some strains produce toxins (pyrogenic exotoxins A, B, and C) that cause a scarlatiniform rash.

Which group of streptococci is associated with erythrogenic toxin production? A. Group A B. Group B C. Group C D. Group G

D Helicobacter pylori is found in specimens from gastric secretions and biopsies and has been implicated as a cause of gastric ulcers. It is found only in the mucus-secreting epithelial cells of the stomach. Both H. pylori and C. jejuni are catalase and oxidase positive. However, Helicobacter spp. are urease positive, which differentiates them from Campylobacter spp.

Which group of tests best differentiates Helicobacter pylori from C. jejuni? A. Catalase, oxidase, and Gram stain B. Catalase, oxidase, and nalidixic acid sensitivity C. Catalase, oxidase, and cephalothin sensitivity D. Urease, nitrate, and hippurate hydrolysis

A P. shigelloides is a lactose nonfermenter that will resemble Shigella spp. on MacConkey agar. Both are TSI Alk/Acid and urease negative. Plesiomonas produces indole and Shigella usually causes delayed production of indole. However, Plesiomonas is oxidase positive, whereas Shigella spp. are oxidase negative.

Which is the best rapid test to differentiate Plesiomonas shigelloides from a Shigella species on selective enteric agar? A. Oxidase B. Indole C. TSI D. Urease

C All four media contain malachite green as an inhibitory agent of nonmycobacteria, but Petragnani's medium contains a higher concentration (0.052 g/dL) than Löwenstein-Jensen (0.025 g/dL), Middlebrook 7H10 (0.0025 g/dL), or American Thoracic Society medium (0.02 g/dL). The last is used for normally sterile specimens, such as from CSF and bone marrow.

Which is the most appropriate nonselective medium for recovery of mycobacteria from a heavily contaminated specimen? A. Löwenstein-Jensen agar B. Middlebrook 7H10 agar C. Petragnani's agar D. American Thoracic Society medium

D The best specimen is the suspected food itself. Stool cultures are not useful because B. cereus is part of the normal fecal flora. The suspected food can be the source of food poisoning by B. cereus if 100,000 or greater organisms per gram of infected food are demonstrated.

Which is the specimen of choice for proof of food poisoning by Bacillus cereus? A. Sputum B. Blood C. Stool D. Food

B EIA and MIF tests detect antibodies to Chlamydia trachomatis, but may also detect antibodies that cross react with other organisms, and are performed on serum. Cell culture is time consuming and dependent on the quality of the specimen. PCR is both sensitive and specific and may be performed on urine.

Which is the test of choice for the confirmation of Chlamydia trachomatis infection in urine? A. Enzyme immunoassay antigen testing B. PCR molecular testing C. Culture using McCoy and Hela cells D. Microimmunofluorescence (MIF) test

D B. pertussis is an oxidase-positive, nonmotile gram-negative coccobacillus and appears as small, round colonies resembling droplets of mercury. It is fastidious and does not grow on chocolate or MacConkey agar. However, B. pertussis adapts to blood agar, growing within 3-6 days. This organism is the cause of whooping cough, which can be prevented by immunization with diphtheria, tetanus, pertussis (DPT) vaccine. The DPT vaccine contains diphtheria and tetanus toxoids and killed whole-cell B. pertussis.

Which media should be used to recover Bordetella pertussis from a nasopharyngeal specimen? A. Chocolate agar B. Blood agar C. MacConkey agar D. Bordet-Gengou agar

D L. pneumophila should be recovered on buffered CYE agar. This agar is nonselective, but can be made more selective for Legionella spp. by addition of the antibiotics cefamandole, polymixin B, and anisomycin. Any small, glistening, convex colonies on buffered CYE agar after 2-3 days of incubation that do not grow on L-cysteine-deficient buffered CYE agar or routine nonselective media should be further tested by the direct fluorescent antibody test (DFA) for confirmation of L. pneumophila.

Which medium is best for recovery of Legionella pneumophila from clinical specimens? A. Chocolate agar B. Bordet-Gengou agar C. New yeast extract agar D. Buffered charcoal-yeast extract (CYE) agar

A Although blood agar will support the growth of Brucella spp., Castenada bottles are the medium of choice. Castenada bottles contain a slant of enriched agar medium that is partially submerged and surrounded by an enriched broth medium. As the specimen is injected into the bottles and mixed, the agar slant is simultaneously coated with the blood (or bone marrow). Brucella is the cause of undulant fever and is responsible for many cases of fever of unknown origin. Brucella spp. are facultative intracellular organisms and grow very slowly, usually requiring 4-6 weeks for recovery. Brucella melitensis is the most frquently recovered species.

Which medium is recommended for the recovery of Brucella spp. from blood and bone marrow specimens? A. Biphasic Castenada bottles with Brucella broth B. Blood culture bottles with Brucella broth C. Bordet-Gengou agar plates and THIO broth D. Blood culture bottles with THIO broth

B M. paratuberculosis is known to cause an ulcerative intestinal disease with chronic diarrhea in cattle known as Johne's disease. While M. paratuberculosis has been isolated from the intestines of humans with Crohn's disease, the organism has not yet been proven to cause ileitis in humans.

Which mycobacterium is associated with Crohn's disease? A. M. marinum B. M. paratuberculosis C. M. avium D. M. gordonae

B M. microti is found in guinea pigs, rabbits, cats, and other warm-blooded animals. It is a cause of tuberculosis in immunocompromised humans, but is also known to cause tuberculosis in immunocompetent persons as well. M. microti cannot be cultured in vitro, but can be distinguished from M. leprae by its distinctive "croissant-like" morphology. Diagnosis is confirmed by NAT.

Which mycobacterium of the M. tuberculosis complex fails to grow in culture and has a characteristic "croissant-like" morphology in stained smears? A. M. africanum B. M. microti C. M. bovis D. M. leprae

D M. gordonae is a nonpathogen, scotochromogen, and rapid grower (7 days at 37°C). Rarely, it is implicated in opportunistic infections in patients with shunts, prosthetic heart values, or hepatoperitoneal disease. The other three species are pathogenic mycobacteria.

Which nonpathogenic Mycobacterium specie is isolated most often from clinical specimens and is called the "tapwater bacillus"? A. M. kansasii B. M. avium-intracellulare complex C. M. leprae D. M. gordonae

A M. leprae and M. microti are different from all other mycobacteria because they cannot be cultured in vitro. Biopsies from nodules and plaques of skin that show numerous AFB are presumptively diagnosed as positive for M. leprae. Confirmatory identification is made by nucleic acid testing (NAT) using PCR.

Which of the following Mycobacterium species is diagnosed by means other than culture? A. M. leprae B. M. bovis C. M. canetti D. M. avium

A M. tuberculosis does not produce pigmentation in the dark or after exposure to light (photochromogen). A common tapwater scotochromogen is Mycobacterium gordonae. The pathogenic scotochromogens are Mycobacterium szulgai, Mycobacterium scrofulaceum, and Mycobacterium xenopi. M. kansasii is a photochromogen producing a yellow pigment following exposure to light and red β-carotene crystals after long incubation periods.

Which of the following Mycobacterium spp. produce(s) pigmented colonies in the dark (is a scotochromogen)? A. M. szulgai B. M. kansasii C. M. tuberculosis D. All of these options

C The hydrolysis of Tween 80 is usually positive when testing the clinically insignificant mycobacteria. M. fortuitum, M. chelonae, and M. gordonae are saprophytic (and opportunistic) species, but M. kansasii is a pathogen. M. kansasii hydrolyses Tween 80 more rapidly than the other species (within 3-6 hours). A positive reaction is indicated by a change in the color of neutral red from yellow to pink.

Which of the following Mycobacterium spp. is best differentiated by the rapid hydrolysis of Tween 80? A. M. fortuitum B. M. chelonae C. M. kansasii D. M. gordonae

A Mycobacteria growing on MacConkey agar are usually nonpathogens. M. chelonae and M. fortuitum are both nonpathogenic rapid growers that will grow on MacConkey agar (with no crystal violet) within 5 days. MAI complex is variable on MacConkey agar but takes much longer to grow. M. marinum and M. ulcerans do not grow on MacConkey agar.

Which of the following Mycobacterium spp. would be most likely to grow on a MacConkey agar plate? A. M. chelonae-fortuitum complex B. M. ulcerans C. M. marinum D. M. avium-intracellulare complex

C F. tularensis is a fastidious gram-negative rod that is best recovered from lymph node aspirates and tissue biopsies. It is oxidase negative, nonmotile, and inert biochemically. Cysteine blood agar is the medium of choice, but F. tularensis will grow on commercially prepared chocolate agar because it contains X factor and is supplemented with a growth enrichment (IsoVitaleX) that contains cysteine. F. tularensis may not grow well on MacConkey agar.

Which of the following amino acids are required for growth of Francisella tularensis? A. Leucine and ornithine B. Arginine and lysine C. Cysteine and cystine D. Histidine and tryptophan

D The first-line antibiotics, except for ethambutol, are bactericidal. Second-line antibiotics used to treat first-line drug-resistant tuberculosis include streptomycin, pyrazinamide, cycloserine, ethionamide, kanamycin, amikacin, viomycin, and capreomycin.

Which of the following drugs are first-line antibiotics used to treat classic tuberculosis for which susceptibility testing is performed by the disk diffusion method on Middlebrook 7H10 or 7H11 agar plates? A. Ampicillin, penicillin, streptomycin, and carbenicillin B. Ampicillin, penicillin, and methicillin C. Vancomycin, methicillin, and carbenicillin D. Isonicotinic acid hydrazide (INH), rifampin, ethambutol, and pyrazinamide

B In addition to coagulase, the virulence of S. aureus is attributed to hyaluronidase, which damages the intercellular matrix (basement membrane) of tissues. β-Lactamase-producing strains are able to inactivate penicillin and ampicillin, making the organism resistant to these antibiotics. Lecithinase is not produced by S. aureus, and urease is not a virulence factor.

Which of the following enzymes contribute to the virulence of S. aureus? A. Urease and lecithinase B. Hyaluronidase and β-lactamase C. Lecithinase and catalase D. Cytochrome oxidase

A Roughly 20% of H. influenzae strains produce β-lactamase, which hydrolyses and inactivates the β-lactam ring of ampicillin (and penicillin).

Which of the following is a characteristic of strains of Haemophilus influenzae that are resistant to ampicillin? A. Production of β-lactamase enzymes B. Hydrolysis of chloramphenicol C. Hydrolysis of urea D. All of these options

A The "whiff" test is used for a presumptive diagnosis of an infection with G. vaginalis. A fishlike odor is noted after the addition of 1 drop of 10% KOH to the vaginal washings. This odor results from the high concentration of amines found in women with vaginosis caused by G. vaginalis.

Which of the following is the best, rapid, noncultural test to perform when Gardnerella vaginalis is suspected in a patient with vaginosis? A. 10% KOH test B. 3% H2O2 test C. 30% H2O2 test D. All of these options

A KV allows the growth of Bacteroides spp., Prevotella spp., and Fusobacterium spp. and inhibits most facultative anaerobic gram-negative rods and gram-positive bacteria (both aerobic and anaerobic). PEA inhibits facultative gram-negative bacteria but will support gram-positive aerobes and anaerobes and gram-negative obligate anaerobes. CCFA is selective for C. difficile from stool, while THIO broth supports gram-positive and gram-negative aerobes and anaerobes.

Which of the following is the medium of choice for the selective recovery of gram-negative anaerobes? A. Kanamycin-vancomycin (KV) agar B. Phenylethyl alcohol (PEA) agar C. Cycloserine-cefoxitin-fructose agar (CCFA) D. THIO broth

C The Eh is most affected by pH and is expressed at pH 7.0. In cultivating anaerobic bacteria, reducing agents such as thioglycollate and L-cysteine are added to anaerobic transport and culture media in order to maintain a low Eh. Certain anaerobes do not grow in the media above a specific critical Eh level.

Which of the following most affects the oxidation-reduction potential (Eh or redox potential) of media for anaerobic bacteria? A. O2 B. Nitrogen C. pH D. Glucose

D Ehrlichia and Anaplasma spp. are intracellular bacteria and are the causative agents of human monocytotropic ehrlichiosis (HME) and human granulocytotropic anaplasmosis (HGA). The organisms infect humans, dogs, and cattle through the bite of an infected Ixodes tick which also transmits Borrelia spp., causing Lyme disease. Blood, CSF, and serum are used for serological testing or PCR. A diagnosis can be made by direct observation of the basophilic inclusions (morulae) in leukocytes of Giemsa's or Wright's stained blood or buffy coat smears. This occurs in about 20% of HME in which the organisms are present in the monocytes, and more frequently in HGA in which they are found in the granulocytes.

Which of the following organisms are transmitted to animals and humans after a tick bite? A. Leptospira. B. Chlamydia and Mycoplasma spp. C. Neisseria sicca D. Ehrlichia and Anaplasma spp.

A Growth rates of mycobacteria are used along with biochemical tests as an aid to identification. M. fortuitum grows within 3-5 days at 37°C and is used as the control for rapid growers. M. tuberculosis grows in 12-25 days at 37°C and is a control organism for slow growers. In addition to M. fortuitum, M. chelonei is a rapid grower (3-5 days at 28°C-35°C). In addition to M. tuberculosis, M. avium and M. kansasii are slow growers (10-21 days at 37°C).

Which of the following organisms are used as controls for rapid growers and slow growers? A. Mycobacterium fortuitum and Mycobacterium tuberculosis B. Mycobacterium avium-intracellulare complex and Mycobacterium tuberculosis C. Mycobacterium chelonei and Mycobacterium fortuitum D. Mycobacterium kansasii and Mycobacterium tuberculosis

D Lyme disease may result in acute arthritis and meningitis and is caused by B. burgdorferi. This spirochete is carried by the deer tick belonging to the Ixodes genus (I. dammini in the Eastern and North-central United States and I. pacificus in the Northwest United States). The life cycle of the tick involves small rodents such as the white-footed mouse and the white-tailed deer.

Which of the following organisms is the cause of Lyme disease? A. Treponema pallidum B. Neisseria meningitidis C. Babesia microti D. Borrelia burgdorferi

D Lipase is produced by some Clostridium spp. and is seen as an iridescent pearly layer on the surface of the colonies that extends onto the surface of the egg yolk agar medium surrounding them. C. perfringens, the most frequently isolated Clostridium species, is negative for lipase production.

Which of the following organisms will display lipase activity on egg yolk agar? A. Clostridium botulinum B. Clostridium sporogenes C. Clostridium novyi (A) D. All of these options

B Production of β-hemolysis is used to distinguish these two species from other Haemophilus with the same X and V requirements. Horse blood agar furnishes X factor and, when supplemented with yeast extract, supports the growth of Haemophilus spp. Sheep blood agar is not used because it contains growth inhibitors for some Haemophilus spp. The chart on the next page summarizes the characteristics of the Haemophilus spp.

Which of the following plates should be used in order to identify Haemophilus haemolyticus and Haemophilus parahaemolyticus? A. Sheep blood agar and chocolate agar B. Horse blood agar and Mueller-Hinton agar with X and V strips C. Brain-heart infusion agar with sheep red cells added D. Chocolate agar and Mueller-Hinton agar with X factor added

A Colonies growing on sheep blood agar secreting NAD (V factor) or producing β-hemolysins (which lyse the sheep RBCs releasing NAD) allow pinpoint-size colonies of Haemophilus spp. to grow around them. Sheep blood agar alone does not support the growth of Haemophilus spp., which require V factor because of the presence of V factor-inactivating enzymes that are present in the agar.

Which of the following products is responsible for satellite growth of Haemophilus spp. around colonies of Staphylococcus and Neisseria spp. on sheep blood agar? A. NAD B. Hemin C. Indole D. Oxidase

D The slide coagulase test using rabbit plasma with ethylenediaminetetraacetic acid (EDTA) detects bound coagulase or "clumping factor" on the surface of the cell wall, which reacts with the fibrinogen in the plasma. This test is not positive for all strains of S. aureus, and a negative result must be confirmed by the tube method for detecting "free coagulase" or extracellular coagulase. The tube test is usually positive within 4 hours at 35°C; however, a negative result must then be incubated at room temperature for the remainder of 18-24 hours. Some strains produce coagulase slowly or produce fibrinolysin, which dissolves the clot at 35°C. Latex agglutination procedures utilize fibrinogen and IgG-coated latex beads that detect protein A on the staphylococcal cell wall.

Which of the following tests is used routinely to identify Staphylococcus aureus? A. Slide coagulase test B. Tube coagulase test C. Latex agglutination D. All of these options

B C. hominis is a gram-negative coccobacillus biochemically similar to Actinobacillus spp. Like Actinobacillus, it is a cause of endocarditis. However, Cardiobacterium spp. are positive for cytochrome oxidase and negative for nitrate reduction, while most Actinobacillus are negative for oxidase and positive for nitrate reduction. C. hominis will grow on blood agar after 48-72 hours in 5% CO2 at 35°C, but Actinobacillus requires chocolate agar.

Which of the following tests may be used to differentiate Cardiobacterium hominis from Actinobacillus spp.? A. Gram stain B. Indole C. Anaerobic incubation D. Oxidase

C Aeromonas hydrophilia and other Aeromonas spp. have been implicated in acute diarrheal disease as well as cellulitis and wound infections. Infections usually follow exposure to contaminated soil, water, or food. Aeromonas growing on enteric media are differentiated from the Enterobacteriaceae by demonstrating that colonies are oxidase positive. The Aeromonas are sometimes overlooked as pathogens because most strains grow on selective enteric agar as lactose fermenters.

Which of the following tests should be done first in order to differentiate Aeromonas spp. from the Enterobacteriaceae? A. Urease B. OF glucose C. Oxidase D. Catalase

A Streptococcus spp. are catalase negative and L. monocytogenes is catalase positive. L. monocytogenes appears on the Gram stain smear as gram-positive short, thin, diphtheroidal shapes, whereas streptococci usually appear as short, gram-positive chains. The reactions shown in the following chart differentiate L. monocytogenes from the group B streptococci.

Which of the following tests should be performed for initial differentiation of Listeria monocytogenes from group B streptococci? A. Gram stain, motility at room temperature, catalase B. Gram stain, CAMP test, H2S/TSI C. Oxidase, CAMP test, glucose D. Oxidase, bacitracin

A The production of acetoin by S. aureus from glucose or pyruvate differentiates it from S. intermedius, which is also coagulase positive. This test is also called the VP test. Acetoin production is detected by addition of 40% KOH and 1% α-naphthol to the VP test broth after 48 hours of incubation. A distinct pink color within 10 minutes denotes a positive test.

Which of the following tests should be used to differentiate Staphylococcus aureus from Staphylococcus intermedius? A. Acetoin B. Catalase C. Slide coagulase test D. Urease

C The modified oxidase test is used to rapidly identify catalase-positive gram-positive cocci as Micrococcus spp. (positive) or Staphylococcus spp. (negative). Filter paper disks that are saturated with oxidase reagent (tetramethyl-p-phenylenediamine in dimethylsulfoxide) are used. A colony of the isolate is rubbed onto the paper. Oxidase-positive organisms produce a purple color within 30 sec.

Which of the following tests will rapidly differentiate micrococci from staphylococci? A. Catalase B. Coagulase C. Modified oxidase D. Novobiocin susceptibility

D P. aeruginosa is highly resistant to many antimicrobial drugs as well as being one of the most often cultured opportunistic organisms. This virulence factor allows for many nosocomial infections such as those of the urinary tract, wounds (burn patients), bacteremia, respiratory tract, and CNS.

Which of the listed Pseudomonas spp. is associated with the following virulence factors: exotoxin A, endotoxins, proteolytic enzymes, antimicrobial resistance, and production of alginate? A. P. fluorescens B. P. putida C. P. stutzeri D. P. aeruginosa

C A test for β-lactamase production should be performed on all isolates of Staphylococcus and Enterococcus as well as N. gonorrhoeae and H. influenzae. Some organisms may appear to be penicillin/ampicillin susceptible by the conventional Kirby-Bauer disk diffusion method for β-lactamase producers, but still produce small amounts of β-lactamase. An example is heteroresistant mecA-positive S. aureus (the mecA gene confers penicillin resistance and is present in all MRSA strains).

Which one of the following organisms is a known producer of β-lactamase-producing strains, and should be tested (screened) by a commercial β-lactamase assay prior to susceptibility testing? A. Streptococcus pneumoniae B. Group B streptococci C. Enterococcus spp. D. Planococcus spp.

A Genital mycoplasmas (M. hominis and Ureaplasma urealyticum) are grown on specific agars. M. hominis is grown on "M" agar containing arginine and phenol red. Colonies of mycoplasma are 50-300 μm in diameter and display a "fried-egg" appearance with red holes. U. urealyticum is isolated from genital specimens on "U" agar (containing urea and phenol red), then subcultured to A7/A8 agar. Colonies of Ureaplasma are small and golden brown on A7/A8 agar.

Which organism typically produces "fried-egg" colonies on agar within 1-5 days of culture from a genital specimen? A. Mycoplasma hominis B. Borrelia burgdorferi C. Leptospira interrogans D. Treponema pallidum

D M. bovis is also called the bovine tubercle bacillus. A nonvirulent strain, bacillus Calmette-Guérin (BCG), is used as a tuberculosis vaccine throughout the world. Infections with M. bovis resemble infections caused by M. tuberculosis and are seen in circumstances where there is close contact between humans and cattle.

Which organism, associated with tuberculosis in cattle, causes tuberculosis in humans, especially in regions where dairy farming is prevalent? A. Mycobacterium avium-intracellulare complex B. Mycobacterium kansasii C. Mycobacterium marinum D. Mycobacterium bovis

A Spore appearance and location, along with Gram stain morphology, aids in distinguishing the Clostridium spp. Round, terminal spores (drumstick spores) are demonstrated when C. tetani is grown in chopped meat with glucose broth. Recognition of spores is particularly important because C. tetani sometimes appears as gram negative.

Which spore type and location is found on Clostridium tetani? A. Round, terminal spores B. Round, subterminal spores C. Ovoid, subterminal spores D. Ovoid, terminal spores

B M. marinum is usually recovered from skin lesions that have been in contact with underchlorinated freshwater, saltwater, or tropical fish aquariums.

Which temperature range is ideal for the recovery of M. marinum? A. 24°C-26°C B. 30°C-32°C C. 42°C-44°C D. 44°C-48°C

A C. botulinum and C. sporogenes have similar characteristics biochemically (see the following chart), and definitive identification of C. botulinum is made by the toxin neutralization test for its neurotoxins in serum or feces. Specimens should be kept at 4°C and sent to the CDC for culture and toxin assays.

Which test is performed in order to confirm an infection with Clostridium botulinum? A. Toxin neutralization B. Spore-forming test C. Lipase test D. Gelatin hydrolysis test

A The cold agglutinin test is nonspecific and time consuming, and therefore, outdated. Although M. pneumonia can be grown on artificial media, many specimens fail to grow, and growth can take up to 3 weeks. CF tests are not as sensitive or specific as EIA tests. Direct antigen assay by immunofluorescence is specific but has a sensitivity at least two orders of magnitude below DNA amplification methods. While PCR is the most sensitive method, it may detect the presence of M. pneumonia in the absence of infection. EIA methods are sensitive, but some persons fail to produce antibodies especially in extra respiratory infections, and antibody tests alone may detect past rather than current infections. Adults may fail to produce an IgM response especially in recurrent infections, so the best confirmatory approach is a combination of EIA and direct antigen testing.

Which test is the most reliable for the detection of Mycoplasma pneumonia in serum and for the confirmation of diagnosis? A. EIA testing and direct antigen testing B. Cold agglutinin testing using Group O RBCs C. Culture on SP4 glucose broth with arginine D. Complement fixation

B By using a 15-μg erythromycin disk adjacent to a 2-μg clindamycin disk in a disk diffusion procedure, a flattening of the clindamycin zone occurs in inducible clindamycin resistance, causing the zone to look like the letter D. This is referred to as being D-zone positive. CLSI recommends that inducible clindamycin resistant strains be reported as resistant with a comment that clindamycin may still be effective in some patients.

Which test is used for the determination of inducible clindamycin resistance in staphylococci and streptococci? A. E-test B. D-zone test C. A-test D. Camp test

D The bile esculin test differentiates those bacteria that can hydrolyze esculin and also grow in the presence of 4% bile salts or 40% bile. The bile esculin slant is inoculated on the surface and incubated for 24-48 hours in a nonCO2 incubator. Group D streptococci (enterococci and nonenterococci) are positive, causing blackening of half or more of the slant within 48 hours. Viridans streptococci are negative (do not grow or hydrolyze esculin).

Which test is used to differentiate the viridans streptococci from the group D streptococci and enterococci? A. Bacitracin disk test B. CAMP test C. Hippurate hydrolysis test D. Bile esculin test

A M. haemophilum, M. ulcerans, and M. marinum grow optimally at 27°C-30°C. M. xenopi grows optimally at 42°C-45°C, and is frequently isolated from hot water systems and storage tanks. All other mycobacterium are cultured at 35°C-37°C.

Which two mycobacteria commonly isolated from subcutaneous skin have an optimal growth temperature of 30°C? A. M. haemophilum and M. ulcerans B. M. kansasii and M. xenopi C. M. gordonae and M. avium D. M. simiae and M. avium

B S. bovis and S. salivarius are physiologically and biochemically similar. They are both PYR and 6.5% salt broth negative and bile esculin positive, but only S. bovis is positive for mannitol and starch reactions. See the following chart.

Which two tests best differentiate S. bovis (group D, nonenterococcus) from Streptococcus salivarius? A. Bile esculin and 6.5% salt broth B. Starch hydrolysis and acid production from mannitol C. Bacitracin and PYR D. Trimethoprim-sulfamethoxazole susceptibility and PYR

C Group A streptococci are sensitive to bacitracin and negative for bile esculin and 6.5% salt broth. Group B streptococci will grow in 6.5% salt broth but are negative for bile esculin and PYR. The nongroup A, B, or D streptococci will not grow in 6.5% salt broth and are sensitive to SXT. Some group D streptococci will hydrolyze hippurate. Enterococcus faecalis is positive for bile esculin, 6.5% salt broth, and PYR.

β-Hemolytic streptococci, more than 50,000 col/mL, were isolated from a urinary tract catheter urine specimen. Given the following reactions, what is the most likely identification? CAMP test = Neg Hippurate hydrolysis = ± Bile solubility = Neg 6.5% salt = + PYR = + Bile esculin = + SXT = Resistant Bacitracin = Resistant Optochin = Resistant A. Group A streptococci B. Group B streptococci C. Enterococcus faecalis D. Nongroup A, nongroup B, nongroup D streptococci

D All species of Acinetobacter are oxidase negative and grow on MacConkey agar. Chryseobacterium spp. produce yellow pigment (like Acinetobacter) but are oxidase positive and do not grow on MacConkey agar. B. cepacia also produces a yellow pigment but is motile.

17. A yellow pigment-producing organism that is oxidase positive, nonmotile, and does not grow on MacConkey agar is: A. Acinetobacter baumannii B. Acinetobacter lwoffii C. Burkholderia cepacia D. Chryseobacterium meningosepticum

D Cold enrichment is especially useful when specimens contain large numbers of normal flora that are sensitive to prolonged exposure to near-freezing temperature. In addition to Yersinia, the technique has been used to enhance recovery of Listeria monocytogenes from specimens containing other bacteria.

21. Cold enrichment of feces (incubation at 4°C) in phosphate-buffered saline prior to subculture onto enteric media enhances the recovery of: A. Enterotoxigenic E. coli B. Salmonella paratyphi C. Hafnia alvei D. Y. enterocolitica

B Proteus spp. are urease positive as are approximately 70% of Y. enterocolitica isolates. Both organisms are lactose negative and motile. However, Yersinia is motile at 22°C and not at 35°C (demonstrated using motility media).

31. Which of the following organisms, found in normal fecal flora, may be mistaken biochemically for the genus Yersinia? A. Klebsiella spp. B. Proteus spp. C. E. coli D. Enterobacter spp.

D Possible pathogenic strains of E. coli should be picked from MacConkey agar and subcultured onto MacConkey agar with sorbitol. After subculture, these strains can be serotyped or sent to a reference laboratory. Most E. coli normal flora ferment D-sorbitol and appear pink to red on MacConkey-sorbitol agar. The E. coli strain O157:H7 causes the enteric disease hemorrhagic colitis. It ferments D-sorbitol slowly or not at all and appears as colorless colonies on MacConkey-sorbitol agar.

32. Why might it be necessary for both pink (lactose-positive) and colorless (lactose-negative) colonies from an initial stool culture on MacConkey agar to be subcultured and tested further for possible pathogens? A. Most Shigella strains are lactose positive B. Most Salmonella strains are maltose negative C. Most Proteus spp. are lactose negative D. Pathogenic E. coli can be lactose positive or lactose negative

A Shigella is the most likely organism biochemically. E. coli are usually indole and motility positive, and Proteus are motility and urease positive. Most Salmonella are H2S positive. Shigella and Campylobacter cause bloody diarrhea because they invade the epithelial cells of the large bowel; however, Campylobacter spp. do not grow on MacConkey agar, and they are oxidase positive.

35. A bloody stool cultured from a 26-year-old woman after 3 days of severe diarrhea showed the following results at 48 hours after being plated on the following media: MacConkey agar: little normal flora with many non-lactose-fermenting colonies Hektoen enteric agar: many blue-green colonies Campylobacter blood agar and C. difficile agar: no growth Clear colonies (from MacConkey agar) tested negative for oxidase, indole, urease, motility, and H2S The most likely identification is: A. Shigella spp. B. Salmonella spp. C. Proteus spp. D. E. coli

C Inflammation with bleeding of the mucosa of the large intestine (hemorrhagic colitis) is a result of an enterohemorrhagic E. coli (EHEC) infection associated with certain serotypes, such as E. coli O157:H7. The source of the E. coli infection is from ingestion of undercooked ground beef contaminated with fecal matter or drinking raw milk.

A 6-year-old female patient was admitted to the hospital following 2 days of severe diarrhea. Cultures from three consecutive stool samples contained blood and mucus. Patient history revealed a hamburger lunch at a fast-food restaurant 3 days earlier. Which pathogen is most likely responsible for the following results? Growth on: XLD agar = yellow colonies HE agar = yellow colonies Mac agar = light pink and dark pink colonies Mac with sorbitol agar - few dark pink and many colorless colonies A. Salmonella spp. B. Shigella spp. C. E. coli O157:H7 D. Yersinia enterocolitica

A S. putrefaciens produces abundant H2S on KIA or TSI. Shewanellae are the only nonfermenters that produce H2S on these media.

A culture from an intra-abdominal abscess produced orange-tan colonies on blood agar that gave the following results: Oxidase = + Nitrate reduction = + KIA = Alk/Alk (H2S)+ Motility = + (single polar flagellum) DNase = + Ornithine decarboxylase = + Growth at 42°C = Neg The most likely identification is: A. Shewanella putrefaciens B. Acinetobacter spp. C. Pseudomonas aeruginosa D. Chryseobacterium spp.

C (C. diphtheriae must be recovered from the deep layers of the pseudomembrane that forms in the nasopharyngeal area. A flexible calcium alginate nasopharyngeal swab is the best choice for collecting a specimen from the posterior nares and pharynx.)

A flexible calcium alginate nasopharyngeal swab is the collection device of choice for recovery of which organism from the nasopharynx? A. Staphylococcus aureus B. Streptococcus pneumoniae C. Corynebacterium diphtheriae D. Bacteroides fragilis

D Moraxella spp. are oxidase positive and nonmotile, which distinguishes them from Acinetobacter spp. and most Pseudomonas spp. Moraxella spp. are highly sensitive to penicillin, but Acinetobacter spp. and Pseudomonas spp. are penicillin resistant. M. lacunata is implicated in infections involving immunosuppressed patients.

A gram-negative coccobacillus was recovered on chocolate agar from the CSF of an immunosuppressed patient. The organism was nonmotile and positive for indophenol oxidase but failed to grow on MacConkey agar. The organism was highly susceptible to penicillin. The most probable identification is: A. Acinetobacter spp. B. Pseudomonas aeruginosa C. Pseudomonas stutzeri D. Moraxella lacunata

B Morganella are biochemically similar to Proteus spp., both being lactose negative, motile, and positive for phenylalanine deaminase and urease. However, Morganella can be differentiated from Proteus spp. based upon H2S, indole, ornithine decarboxylase, and xylose fermentation. Ewingella spp. are usually positive (70%) for lactose fermentation, whereas the other three genera are lactose negative.

A gram-negative rod is recovered from a catheterized urine sample from a nursing home patient. The lactose-negative isolate tested positive for indole, urease, ornithine decarboxylase, and phenylalanine deaminase and negative for H2S. The most probable identification is: A. Edwardsiella spp. B. Morganella spp. C. Ewingella spp. D. Shigella spp.

D K. pneumoniae and E. cloacae display similar IMViC (indole, MR, VP, and citrate) reactions (00++) and TSI results. However, approximately 65% of E. cloacae strains are urease positive compared with 98% of those of K. pneumoniae. Enterobacter spp. are motile and Klebsiella are nonmotile. The antibiotic pattern of resistance to carbenicillin and ampicillin is characteristic for Klebsiella.

A leg culture from a nursing home patient grew gram-negative rods on MacConkey agar as pink to dark pink oxidase-negative colonies. Given the following results, which is the most likely organism? TSI = A/A Indole = Neg MR = Neg VP = + Citrate = + H2S = Neg Urease = + Motility = Neg Antibiotic susceptibility: resistant to carbenicillin and ampicillin A. Serratia marcescens B. Proteus vulgaris C. Enterobacter cloacae D. Klebsiella pneumoniae

B S. marcescens has been implicated in numerous nosocomial infections and is recognized as an important pathogen with invasive properties. Gelatin hydrolysis and DNase are positive for both the Proteusspp. and Serratia, but the negative urease and phenylalanine deaminase are differential. E. cloacae does not produce DNase, gelatinase, or lysine decarboxylase.

A leg-wound culture from a hospitalized 70-year-old diabetic man grew motile, lactose-negative colonies on MacConkey agar. Given the following biochemical reactions at 24 hours, what is the most probable organism? H2S (TSI) = Neg Indole = Neg MR = Neg VP = + DNase = + Citrate = + Urease = Neg Phenylalanine deaminase = Neg Ornithine and lysine decarboxylase = + Arginine decarboxylase = Neg Gelatin hydrolysis = + A. Proteus vulgaris B. Serratia marcescens C. Proteus mirabilis D. Enterobacter cloacae

A P. stutzeri produces dry, wrinkled colonies that are tough and adhere to the media as well as smooth colonies. B. pseudomallei produces similar colony types but is distinguished by biochemical tests and susceptibility to the polymyxins. The colonies of P. stutzeri are buff to light brown because of the relatively high concentration of cytochromes.

A nonfermenter recovered from an eye wound is oxidase positive, motile with polar monotrichous flagella, and grows at 42°C. Colonies are dry, wrinkled or smooth, buff to light brown in color, and are difficult to remove from the agar. In which DNA homology group should this organism be placed? A. Pseudomonas stutzeri B. Pseudomonas fluorescens C. Pseudomonas alcaligenes D. Pseudomonas diminuta

A The Simmons citrate test determines if an organism can utilize citrate as the sole source of carbon. The medium turns blue, indicating the presence of alkaline products such as carbonate. Tubes are incubated a minimum of 24 hours at 35°C with a loose cap before reading.

A positive Simmons citrate test is seen as a: A. Blue color in the medium after 24 hours of incubation at 35°C B. Red color in the medium after 18 hours of incubation at 35°C C. Yellow color in the medium after 24 hours of incubation at 35°C D. Green color in the medium after 18 hours of incubation at 35°C

A V. cholerae and C. difficile are usually not included in a routine stool culture. If Vibrio spp. are suspected, a special request should be included. Although MacConkey agar will support the growth of Vibrio spp., normal enteric flora overgrow and occlude these organisms. C. difficile culture requires special media (CCFA) that inhibit other anaerobic flora and facultative anaerobic flora and should be requested specifically if symptoms warrant. MacConkey agar with sorbitol will allow the E. coli O157:H7 to be recovered. Yersinia spp. can be detected on a regular MacConkey agar plate.

A routine, complete stool culture procedure should include media for the isolation of E. coli O157:H7 as well as: A. Salmonella, Shigella, Yersinia, Campylobacter, and Staphylococcus aureus B. Vibrio cholerae, Brucella, and Yersinia spp. C. S. aureus, group B streptococci, and group D streptococci D. Clostridium difficile, Clostridium perfringens, and Yersinia spp.

B (Both gram-positive cocci and gram-negative bacilli will grow on blood agar plates, but the medium is used in conjunction with a selective medium such as CNA agar for gram-positive cocci and MacConkey agar for gram-negative bacilli. H. influenzae requires X and V factors, and H. parainfluenzae requires V factor; the primary isolation medium for Haemophilus is chocolate agar.)

A sheep blood agar plate is used as a primary isolation medium when all of the following organisms are to be recovered from a wound specimen except: A. β-Hemolytic streptococci and coagulase-positive staphylococci B. Haemophilus influenzae and Haemophilus parainfluenzae C. Proteus spp. and Escherichia coli D. Pseudomonas spp. and Acinetobacter spp.

C P. aeruginosa is a cause of a significant number of burn wound infections; these organisms can exist in distilled water and underchlorinated water. Acinetobacter spp. are oxidase negative and Moraxella spp. are highly susceptible to penicillin, ruling them out as possible causes.

A specimen from a 15-year-old female burn patient was cultured after débridement, and the following results were obtained: Oxidase = + Lysine decarboxylase = Neg Catalase = + Motility = + Ornithine decarboxylase Glucose = + for oxidation = Neg (open tube) Arginine dihydrolase = + Maltose = Neg for oxidation (open tube) Penicillin = Resistant Aminoglycosides = Susceptible Colistin (Polymixin B) = Susceptible These results indicate which of the following organisms? A. Acinetobacter baumannii B. Moraxella lacunata C. Pseudomonas aeruginosa D. Acinetobacter lwoffii

A The most likely organism is a species of Shigella. Typically, Salmonella spp. produce H2S-positive colonies that display black centers on the differential media (except on MacConkey agar). The biochemical tests listed are necessary to differentiate Shigella from E. coli because some E. coli strains cross-react with Shigella typing sera. Shigella spp. are one of the most common causes of bacterial diarrhea; group D (S. sonnei) and group B (S. flexneri) are the species most often isolated.

A stool culture from a 30-year-old man suffering from bloody mucoid diarrhea gave the following results on differential enteric media: MacConkey agar = clear colonies; XLD agar = clear colonies; Hektoen agar = green colonies; Salmonella-Shigella agar = small, clear colonies Which tests are most appropriate for identification of this enteric pathogen? A. TSI, motility, indole, urease, Shigella typing with polyvalent sera B. TSI, motility, indole, lysine, Salmonella typing with polyvalent sera C. TSI, indole, MR, VP, citrate D. TSI, indole, MR, and urease

D (Yersinia pestis is on the list of agents of bioterrorism. Isolation and identification should be performed in a facility with a Level II or higher biosafety rating. If there is a high risk of aerosolizing the specimen during processing, procedures should be performed under Level III biosafety conditions. Recovery of Y. pestis is highest if the specimen is cultured within 2 hours of collection.)

A suspected case of plague requires which of the following procedures in order to confirm Yersinia pestis? A. Collection of multiple sets of blood cultures B. Incubation of blood cultures at both 28°C and 35°C C. Culture aspirates from bubos to MacConkey agar at room temperature D. All of these options

A (A thin film of growth appearing in the zone area of inhibition around the susceptibility disk should be ignored when swarming Proteus or other organisms are encountered. Discontinuous, poor growth or tiny colonies near the end of the zone should also be ignored.)

According to the Kirby-Bauer standard antimicrobial susceptibility testing method, what should be done when interpreting the zone size of a motile, swarming organism such as a Proteus species? A. The swarming area should be ignored B. The results of the disk diffusion method are invalid C. The swarming area should be measured as the growth boundary D. The isolate should be retested after diluting to a 0.05 McFarland standard

A Alcaligenes and Bordetella are genera belonging to the Alcaligenaceae family. The two organisms are very similar biochemically, but B. bronchiseptica is urease positive. Both organisms are oxidase positive, grow on MacConkey agar, and are motile by peritrichous flagella. B. bronchiseptica grows well on MacConkey agar but other species of Bordetella are fastidious gram-negative rods.

Alcaligenes faecalis (formerly A. odorans) is distinguished from Bordetella bronchiseptica with which test? A. Urease (rapid) B. Oxidase C. Growth on MacConkey agar D. Motility

D (MTM, New York City, and Martin-Lewis agars contain blood factors needed to support the growth of N. gonorrhoeae as well as antibiotics that prevent growth of normal genital flora. Cultures must be incubated in 3%-7% CO2 at 35°C. Cultures should be held a minimum of 48 hours before being considered negative.)

All of the following are appropriate when attempting to isolate N. gonorrhoeae from a genital specimen except: A. Transport the genital swab in charcoal transport medium B. Plate the specimen on modified Thayer-Martin (MTM) medium C. Plate the specimen on New York City or Martin-Lewis agar D. Culture specimens in ambient oxygen at 37°C

A An isolate of E. coli recovered from a stool culture in hemorrhagic colitis can be definitely identified only by serotyping. The isolate is identified as E. coli by the usual biochemical reactions. The strain of E. coli responsible for hemorrhagic colitis is O157:H7 and is usually negative for sorbitol fermentation. Colonies of this strain of E. coli appear colorless on MacConkey agar with sorbitol added.

An isolate of E. coli recovered from the stool of a patient with severe bloody diarrhea should be tested for which sugar before sending it to a reference laboratory for serotyping? A. Sorbitol (fermentation) B. Mannitol (oxidation) C. Raffinose (fermentation) D. Sucrose (fermentation)

A Chryseobacterium spp. and Acinetobacter spp. often produce a yellow pigment on blood or chocolate agar and are nonmotile. Acinetobacter spp. are oxidase negative, grow on MacConkey agar, and are coccobacillary on the Gram stain smear. In contrast, Chryseobacterium spp. are oxidase positive, do not grow on MacConkey agar, and are typically rod shaped. Chryseobacterium meningosepticum is highly pathogenic for premature infants.

Chryseobacterium spp. are easily distinguished from Acinetobacter spp. by which of the following two tests? A. Oxidase, growth on MacConkey agar B. Oxidase and OF (glucose) C. TSI and urea hydrolysis D. TSI and VP

D (Fastidious organisms such as Neisseria and Haemophilus frequently isolated from the CSF of patients with bacterial meningitis are preserved by placing the fluid in 3%-7% CO2 at 35°C-37°C (or at room temperature for no longer than 30 min), if the specimen cannot be cultured immediately.)

Cerebrospinal fluid (CSF) should be cultured immediately, but if delayed the specimen should be: A. Refrigerated at 4°C to 6°C B. Frozen at -20°C C. Stored at room temperature for no longer than 24 hours D. Incubated at 37°C and cultured as soon as possible

B Cetrimide (acetyl trimethyl ammonium bromide) agar is used for the isolation and identification of P. aeruginosa. With the exception of P. fluorescens, the other pseudomonads are inhibited along with related nonfermentative bacteria.

Cetrimide agar is used as a selective isolation agar for which organism? A. Acinetobacter spp. B. Pseudomonas aeruginosa C. Moraxella spp. D. Stenotrophomonas maltophilia

B (Chocolate agar provides X factor (hemin) and V factor (NAD) required for the growth of Haemophilus spp. Thayer-Martin medium is a chocolate agar containing the antibiotics that permit isolation of N. gonorrhoeae in specimens containing large numbers of gram-negative bacteria, including commensal Neisseria spp.)

Chocolate agar and modified Thayer-Martin agar are used for the recovery of: A. Haemophilus spp. and Neisseria spp., respectively B. Haemophilus spp. and N. gonorrhoeae, respectively C. Neisseria spp. and Streptococcus spp., respectively D. Streptococcus spp. and Staphylococcus spp., respectively

A (DCA inhibits gram-positive organisms. N. gonorrhoeae and Neisseria meningitidis are too fastidious to grow on DCA. Citrate and deoxycholate salts inhibit growth of gram-positive bacteria. The media contain lactose and neutral red, allowing differentiation of lactose fermenters (pink colonies) from nonfermenters (colorless).)

Deoxycholate agar (DCA) is useful for the isolation of: A. Enterobacteriaceae B. Enterococcus spp. C. Staphylococcus spp. D. Neisseria spp.

D Bubonic plague involves an inflammatory swelling of the lymph nodes of the axilla and groin, whereas pneumonic plague is associated with an airborne route involving the lungs. Both infections are caused by the same member of the Enterobacteriaceae family, Yersinia pestis.

Direct spread of pneumonic plague disease occurs by which route? A. Fecal-oral route B. Rat bite C. Ingestion of contaminated tissue D. Inhalation of contaminated airborne droplets

A (Sheep RBCs are used in blood agar plates because they are readily available and less inhibitory than cells of other species. The type of hemolysis is determined by the source of RBCs. Sheep RBCs are chosen because of the characteristically clear hemolysis produced by β-hemolytic streptococci, Staphylococcus, and other pathogens producing β-hemolysins. Sheep blood does not support the growth of Haemophilus haemolyticus, eliminating the possibility of confusing it with β-hemolytic streptococci in throat cultures.)

In the United States, most blood agar plates are prepared with 5% or 10% red blood cells (RBCs) obtained from: A. Sheep B. Horses C. Humans D. Dogs

B The test for urease production is based on the ability of the colonies to hydrolyze urea in Stuart broth or Christensen agar to form CO2 and ammonia. These products form ammonium carbonate, resulting in alkalinization. This turns the pH indicator (phenol red) pink at pH 8.0.

In the test for urease production, ammonia reacts to form which product? A. Ammonium citrate B. Ammonium carbonate C. Ammonium oxalate D. Ammonium nitrate

A Approximately 15 cases of Y. pestis infection are confirmed in the United States annually. Most originate in the Southwest. It is necessary to be aware of this regional occurrence because untreated cases are associated with a mortality rate of approximately 60%. Y. pestis is not fastidious and grows well on blood agar. It is inactive biochemically, which helps to differentiate it from other Enterobacteriaceae.

Infections caused by Yersinia pestis are rare in the United States. Those cases that do occur are most frequently located in which region? A. New Mexico, Arizona, and California B. Alaska, Oregon, and Utah C. North and South Carolina and Virginia D. Ohio, Michigan, and Indiana

D DNase is not produced by Cedecea spp. but is produced (along with proteinases) by Serratia spp. Other key differential tests include lipase (positive for Cedecea, negative for Serratia) and gelatin hydrolysis (negative for Cedecea, positive for Serratia).

Members of the genus Cedecea are best differentiated from Serratia spp. by which test result? A. Positive motility B. Positive urease C. Positive phenylalanine deaminase D. Negative DNase

A The use of OF tubes helps to determine the presumption of a nonfermentative bacillus (glucose oxidation positive and glucose fermentation negative). The positive cytochrome oxidase test and pigment production indicate a possible Pseudomonas species. Several NFB produce pigments that aid in species identification: P. aeruginosa produces yellow pyoverdins (fluorescein) and/or pyocyanin (blue aqua pigment). The characteristic grapelike odor of aminoacetophenone as well as growth at 42°C are characteristics of P. aeruginosa.

Presumptive tests used for identification of the Pseudomonas spp. are: A. Oxidase, oxidation-fermentation (OF) glucose (open), OF glucose (sealed), motility, pigment production B. Growth on blood agar plate (BAP) and eosin-methylene blue (EMB) agars, lysine eudomonas species. Several NFB produce pigments that aid in species identificationdecarboxylation, catalase C. Growth on MacConkey, EMB, and XLD agars and motility D. Growth on mannitol salt agar and flagellar stain

A (The oxidase positive test result rules out the members of the Enterobacteriaceae family. Colonies of Aeromonas hydrophilia and Plesiomonas spp. might be mistaken for Vibrio spp. since all three grow as clear colonies on MacConkey agar, are beta hemolytic on blood agar, and are oxidase positive.)

SITUATION: Abdominal pain, fever, vomiting, and nausea prompted an elderly male to seek medical attention. A watery stool specimen producing no fecal leukocytes or erythrocytes was cultured and grew a predominance of gram-negative fermentative bacilli. The colonies were beta-hemolytic on blood agar and cream colored on MacConkey agar. The colonies were both oxidase and catalase positive. What is the most likely identification? A. Aeromonas hydrophilia B. Escherichia coli C. Salmonella spp. D. Shigella spp.

A (All three organisms are positive for oxidase production and are motile. Plesiomonas spp. do not grow on TCBS agar. Clear colonies on MacConkey agar and yellow colonies on TCBS agar indicate Vibrio or Aeromonas spp. However, only Vibrio spp. require Na+ (1% NaCl) in the medium for growth.)

SITUATION: Several attendees of a medical conference in the Gulf coast area became ill after frequenting a seafood restaurant. A presumptive identification of Vibrio cholera was made after stool specimens from several subjects grew clear colonies on MacConkey agar and yellow colonies on TCBS agar. Which key tests would help eliminate Aeromonas and Plesiomonas spp.? A. Mannitol fermentation, Na+ requirement B. Oxidase, motility C. Oxidase, nitrate D. Hemolysis on blood agar, catalase

B (Hektoen agar selectively isolates pathogenic coliforms, especially Salmonella and Shigella. MacConkey agar differentiates lactose fermenters from nonfermenters. CNA agar contains antibiotics that prohibit growth of gram-negative coliforms but not gram-positive cocci. Campy agar contains the antibiotics cephalothin, trimethoprim, vancomycin, polymyxin B, and amphotericin B to prevent growth of Enterobacteriaceae, Pseudomonas spp., and fungi.)

Select the primary and differential media of choice for recovery of most fecal pathogens. A. MacConkey, blood, birdseed, and Campylobacter (Campy) agars B. Hektoen, MacConkey, Campy, colistin-nalidixic acid (CNA) agars C. CNA and Christensen urea agars and thioglycollate media D. Blood, Campy, Mueller-Hinton agars, and thioglycollate media

A (Specimens for culture of N. gonorrhoeae are best if plated immediately or transported in a medium containing activated charcoal to absorb inhibitory substances that hinder their recovery.)

Semisolid transport media such as Amies, Stuart, or Cary-Blair are suitable for the transport of swabs for culture of most pathogens except: A. Neisseria gonorrhoeae B. Enterobacteriaceae C. Campylobacter fetus D. Streptococcus pneumoniae

A Acetoin or carbinol, an end product of glucose fermentation, is converted to diacetyl after the addition of the VP reagents (α-naphthol and 40% potassium hydroxide [KOH]). Diacetyl is seen as a red- to pink-colored complex.

The Voges-Proskauer (VP) test detects which end product of glucose fermentation? A. Acetoin B. Nitrite C. Acetic acid D. Hydrogen sulfide

B (In order to attain asepsis of the skin, 70% alcohol followed by 2% iodine is used for obtaining blood cultures.)

The aseptic collection of blood cultures requires that the skin be cleansed with: A. 2% iodine and then 70% alcohol solution B. 70% alcohol and then 2% iodine or an iodophor C. 70% alcohol and then 95% alcohol D. 95% alcohol only

D (The breakpoint refers to an antimicrobial concentration in the serum associated with optimal therapy using the customary dosing schedule. An organism is susceptible if the MIC is at or below the breakpoint.)

The breakpoint of an antimicrobial drug refers to: A. The amount needed to cause bacteriostasis B. A minimum inhibitory concentration (MIC) of 16 μg/mL or greater C. A MIC of 64 μg/mL or greater D. The level of drug that is achievable in serum

A The drugs of choice for the Enterobacteriaceae vary, and several genera display patterns of resistance that aid in their identification. K. pneumoniae and Citrobacter diversus are resistant to ampicillin and carbenicillin; most Enterobacter spp. and Hafnia are resistant to ampicillin and cephalothin. Proteus, Morganella, and Serratia are resistant to colistin. Providencia and Serratia are resistant to multiple drugs. Several genera are resistant to chloramphenicol and most are resistant to penicillin.

The drugs of choice for treatment of infections with Enterobacteriaceae are: A. Aminoglycosides, trimethoprim-sulfamethoxazole, third-generation cephalosporins B. Ampicillin and nalidixic acid C. Streptomycin and isoniazid D. Chloramphenicol, ampicillin, and colistin

D The oxidase test and red pigment (pyorubin), as well as growth at 42°C, distinguish P. aeruginosa from the other pseudomonads listed, particularly B. cepacia, which is also associated with cystic fibrosis.

The following results were obtained from a pure culture of gram-negative rods recovered from the pulmonary secretions of a 10-year-old cystic fibrosis patient with pneumonia: Oxidase = + Motility = + Glucose OF (open) = + Gelatin hydrolysis = + Pigment = Red Arginine dihydrolase = + (nonfluorescent) Growth at 42°C = + Flagella = + (polar, monotrichous) Which is the most likely organism? A. Burkholderia pseudomallei B. Pseudomonas stutzeri C. Burkholderia cepacia D. Pseudomonas aeruginosa

B Preliminary serological grouping of the Salmonella spp. and Shigella spp. should be performed, since reliable commercial polyvalent antisera are available. Sorbitol-negative (MacConkey agar with sorbitol) colonies of E. coli should be tested using commercially available antisera for somatic "O" antigen 157 and flagellar "H" antigen 7. However, Yersinia pestis isolates should be sent to a public health laboratory for testing, since clinical laboratories generally do not have the typing sera available.

The majority of clinical laboratories with a microbiology department should have the capability of serotyping which pathogenic Enterobacteriaceae? A. Yersinia enterocolitica, Shigella spp. B. E. coli O157:H7, Salmonella spp., Shigella spp. C. Yersinia pestis, Salmonella spp. D. Edwardsiella spp., Salmonella spp.

C The malonate test determines whether an organism can utilize sodium malonate as the sole source of carbon. Malonate is broken down, forming alkaline metabolites that raise the pH of the broth above 7.6. This causes bromthymol blue to turn from green to deep blue (Prussian blue). E. coli, Shigella, and most Salmonella are malonate negative, whereas Enterobacter and Salmonella (formerly Arizona) subgroups 2, 3a, and 3b are positive. Proteus, Providencia, Serratia, and Yersinia are also malonate negative.

The malonate test is most useful in differentiating which members of the Enterobacteriaceae? A. Shigella B. Proteus C. Salmonella subgroups 2, 3 (the former Arizona) D. Serratia

C The two genera, Pseudomonas and Stenotrophomonas, are motile and grow well on MacConkey agar. However, P. aeruginosa is oxidase positive and grows at 42°C but is motile only by polar monotrichous flagella. S. maltophilia is oxidase negative, does not grow at 42°C, and is motile by a polar tuft of flagella.

The most noted differences between P. aeruginosa and Stenotrophomonas maltophilia are: A. Oxidase, catalase, and TSI B. Oxidase, catalase, and ONPG C. Oxidase, 42°C growth, and polar tuft of flagella D. Catalase, TSI, and pigment

A (β-Lactamase production by bacteria that are resistant to penicillin and cephalosporin is detected using one of these drugs as a substrate. Penicillin is hydrolyzed by β-lactamase into acidic products that can be detected as a color change by a pH indicator. In the iodometric method, a disk containing a penicillin-starch substrate turns blue when a drop of iodine is added. A loop of β-lactamase-positive organisms applied to the center of the blue spot will reduce the iodine to iodide, causing the area to clear. The most sensitive method of detection is based upon the ability of the organism to hydrolyze the β-lactam ring of a chromogenic cephalosporin.)

The most sensitive method for the detection of β-lactamase in bacteria is by the use of: A. Chromogenic cephalosporin B. Penicillin C. Oxidase D. Chloramphenicol acetyltransferase

B The ONPG test detects β-galactosidase activity and is most useful in distinguishing late lactose fermenters from lactose nonfermenters. Some strains of E. coli are slow lactose fermenters and may be confused with Shigella spp., which do not ferment lactose. E. coli are ONPG positive while Shigella spp. are ONPG negative.

The ortho-nitrophenyl-β-galactopyranoside (ONPG) test is most useful when differentiating: A. Salmonella spp. from Pseudomonas spp. B. Shigella spp. from some strains of Escherichia coli C. Klebsiella spp. from Enterobacter spp. D. Proteus vulgaris from Salmonella spp.

A Although P. mirabilis is more frequently recovered from patients with urinary tract infections, P. vulgaris is commonly recovered from immunosuppressed patients. P. mirabilis is indole negative and ornithine decarboxylase positive but otherwise is very similar to P. vulgaris.

Three blood cultures taken from a 30-year-old cancer patient receiving chemotherapy and admitted with a urinary tract infection grew lactose-negative, motile, gram-negative rods prior to antibiotic therapy. Given the following biochemical reactions, which is the most likely organism? H2S (TSI) = + Indole = + MR = + VP = Neg Citrate = Neg Urease = + DNase = + Phenylalanine deaminase = + Gelatin hydrolysis = + Ornithine decarboxylase = Neg A. Proteus vulgaris B. Proteus mirabilis C. Serratia marcescens D. Klebsiella pneumoniae

C Most members of the Enterobacteriaceae family produce detectable growth on MacConkey agar within 24 hours. Yersinia enterocolita produces non-lactose-fermenting colonies on MacConkey agar, salmon-colored colonies on Hektoen agar, and yellow or colorless colonies on XLD agar. If Yersinia enterocolitica is suspected, specialized agar (CIN) is employed. The typical bulls-eye colonies, dark red with a translucent border, can be confused with Aeromonas spp. that appear similarly on CIN agar. To differentiate, an oxidase test must be performed, since Yersinia spp. are oxidase negative and Aeromonas spp. are oxidase positive.

Three consecutive stool cultures from a 25-year-old male patient produced scant normal fecal flora on MacConkey and Hektoen agars. However, colonies on CIN agar (cefsulodin-irgasan-novobiocin) displayed "bulls-eye" colonies after 48 hours incubation. The patient had been suffering from enterocolitis with fever, diarrhea, and abdominal pain for 2 days. What is the most likely identification of this gram-negative rod? A. E. coli B. Proteus mirabilis C. Yersinia enterocolitica D. Klebsiella pneumoniae

B Phenylalanine deaminase oxidatively deaminates phenylalanine, forming phenylpyruvic acid. When a solution of ferric chloride is added, the iron reacts with phenylpyruvic acid, forming a green-colored complex. Phenylalanine deaminase is found in the genera Morganella, Providencia, and Proteus and is an excellent test to determine if an organism belongs to this group. Rarely, isolates of Enterobacter may be phenylalanine deaminase positive as well.

Which genera are positive for phenylalanine deaminase? A. Enterobacter, Escherichia, and Salmonella B. Morganella, Providencia, and Proteus C. Klebsiella and Enterobacter D. Proteus, Escherichia, and Shigella

A Shigella spp. and Klebsiella spp. are for the most part nonmotile. Yersinia can be motile at 22°C but is nonmotile at 36°C. Other members of the Enterobacteriaceae that have been isolated from human specimens and are usually nonmotile include Leminorella, Rahnella, and Tatumella.

Which genera of Enterobacteriaceae are usually nonmotile at 36°C? A. Shigella, Klebsiella, and Yersinia B. Escherichia, Edwardsiella, and Enterobacter C. Proteus, Providencia, and Salmonella D. Serratia, Morganella, and Hafnia

B Escherichia, Salmonella, Shigella, and Yersinia are responsible for the majority of enteric diarrhea cases attributable to the Enterobacteriaceae family.

Which genera of the Enterobacteriaceae are known to cause diarrhea and are considered enteric pathogens? A. Enterobacter, Klebsiella, Providencia, and Proteus B. Escherichia, Salmonella, Shigella, and Yersinia C. Pseudomonas, Moraxella, Acinetobacter, and Aeromonas D. Enterobacter, Citrobacter, and Morganella

B

Which group of tests best differentiates Enterobacter aerogenes from Edwardsiella tarda? A. Motility, citrate, and urease B. Hydrogen sulfide (H2S) production, sucrose fermentation, indole, and VP C. Lysine decarboxylase, urease, and arginine dihydrolase D. Motility, H2S production, and DNase

A Morganella and Proteus spp. are motile, produce urease, and deaminate phenylalanine.

Which group of tests best identifies the Morganella and Proteus genera? A. Motility, urease, and phenylalanine deaminase B. Malonate, glucose fermentation, and deoxyribonuclease (DNase) C. Indole, oxidase, MR, and VP D. Indole, citrate, and urease

D All Enterobacteriaceae are oxidase negative. Because E. coli and Proteus spp. comprise a majority of the organisms recovered from clinical specimens, they can be initially identified through rapid testing without additional overnight testing. E. coli display a positive indole test, and the colonial morphology on MacConkey agar is distinctive, showing flat, pink (lactose-positive) colonies with a ring of bile precipitation. Proteus spp. swarm on blood agar and are indole negative.

Which group of tests, along with colonial morphology on primary media, aids most in the rapid identification of the Enterobacteriaceae? A. MR and VP, urease, and blood agar plate B. Phenylalanine deaminase, urease, and CDC agar plate C. Bacitracin, β-lactamase, and MacConkey agar plate D. Indole, oxidase, MacConkey, and blood agar plates

A Point mutations occur in most members of the Enterobacteriaceae family that result in production of a β-lactamase that hydrolyzes broad-spectrum antibiotics such as the cephalosporins as well as penicillin and monobactam antibiotics. These are known as ESBL producers. The most common ESBL organisms are Klebsiella pneumonia and E. coli. ESBL strains are detected by demonstrating their resistance to β-lactam antibiotics.

Which isolates of the Enterobacteriaceae family most commonly produce extended-spectrum β-lactamase (ESBL)? A. E. coli and Klebsiella pneumoniae B. Yersinia enterocolitica and Yersinia pestis C. Morganella morganii and Proteus vulgaris D. Salmonella typhi and Shigella sonnei

D Enterobacteriaceae are grouped according to their ability to ferment lactose, a β-galactoside. Salmonella, Shigella, Proteus, Providencia, and Morganella are usually lactose nonfermenters. Others—including certain strains of E. coli, S. sonnei, Hafnia alvei, Serratia marcescens, and some Yersinia—appear to be lactose nonfermenters because they lack the permease enzyme that actively transports lactose across the cell membrane. However, true lactose nonfermenters do not possess β-galactosidase. The test for β-galactosidase uses the substrate o-nitrophenyl-β-galactopyranoside. At an alkaline pH, β-galactosidase hydrolyses the substrate, forming o-nitrophenol, which turns the medium yellow.

Which of the following organisms are generally positive for β-galactosidase? A. Salmonella spp. B. Shigella spp. C. Proteus spp. D. E. coli

B Biochemical differentiation is essential because Citrobacter isolates may give a false-positive agglutination test with Salmonella grouping sera. C. freundii strains, like Salmonella spp., are usually H2S producers and may be confused with Salmonella spp. unless the proper biochemical tests are utilized. C. freundii and Salmonella spp. are adonitol, indole, and malonate negative. However, C. freundii is KCN positive, whereas Salmonella spp. are KCN negative.

Which of the following organisms is often confused with the Salmonella species biochemically and on plated media? A. E. coli B. Citrobacter freundii C. Enterobacter cloacae D. Shigella dysenteriae

A The indole test detects the conversion of tryptophan (present in the media) to indole by the enzyme tryptophanase. Indole is detected by the reaction with the aldehyde group of p-dimethylaminobenzaldehyde (the active reagent in Kovac's and Ehrlich's reagents) in acid, forming a red complex.

Which of the following reagents is added to detect the production of indole? A. p-Dimethylaminobenzaldehyde B. Bromcresol purple C. Methyl red D. Cytochrome oxidase

B E. coli, positive for lactose, indole, and ONPG are usually motile. Shigella species do not ferment lactose or produce indole, lack β-galactosidase, and are nonmotile.

Which of the following tests best differentiates Shigella species from E. coli? A. Hydrogen sulfide, VP, citrate, and urease B. Lactose, indole, ONPG, and motility C. Hydrogen sulfide, MR, citrate, and urease D. Gas, citrate, and VP

D (In vitro testing of drugs is reliable if the method is standardized. In addition to the first three variables, the type of media and the stability of antibiotics affect the results of MIC testing and must be carefully controlled.)

Which of the following variables may change the results of an MIC? A. Inoculum size B. Incubation time C. Growth rate of the bacteria D. All of these options

D B. pseudomallei produces wrinkled colonies resembling P. stutzeri. Infections are usually asymptomatic and can be diagnosed only by serological methods. The organism exists in soil and water in an area of latitude 20° north and south of the equator (mainly in Thailand and Vietnam). Thousands of U.S. military personnel were infected with these bacteria during the 1960s and 1970s. The disease may reactivate many years after exposure and has been called the "Vietnamese time bomb."

Which organism is associated with immunodeficiency syndromes and melioidosis (a glanders-like disease in Southeast Asia and northern Australia)? A. Pseudomonas aeruginosa B. Pseudomonas stutzeri C. Pseudomonas putida D. Burkholderia pseudomallei

A C. freundii and Salmonella spp. are H2S positive and indole, VP, and phenylalanine deaminase negative. Biochemical characteristics that help to differentiate C. freundii from Salmonella include lactose fermentation (50% of C. freundii are lactose positive, whereas 100% of Salmonella are lactose negative) and urease production (70% of Citrobacter are positive and greater than 99% of Salmonella are negative).

Which organism is sometimes mistaken for Salmonella and will agglutinate in Salmonella polyvalent antiserum? A. C. freundii strains B. Proteus mirabilis strains C. S. sonnei strains D. E. coli

A (Chlamydiae are strict intracellular organisms and must be cultured using living cells. Direct smears can also be made at the time of culture. Staining cells with iodine may reveal the characteristic reddish-brown inclusions sometimes seen in Chlamydia infections. Fluorescein-conjugated monoclonal antibodies may be used to identify the organisms in infected cells.)

Which procedure is appropriate for culture of genital specimens in order to recover Chlamydia spp.? A. Inoculate cycloheximide-treated McCoy cells B. Plate onto blood and chocolate agar C. Inoculate into thioglycollate (THIO) broth D. Plate onto modified Thayer-Martin agar within 24 hours

A In the nitrate test, nitrites formed by bacterial reduction of nitrates will diazotize sulfanilic acid. The diazonium compound complexes with α-naphthylamine, forming a red product. Media containing nitrates are used for the identification of nonfermenters. When testing nonfermenters, it is wise to confirm a negative reaction using zinc dust. The diazonium compound detects nitrite only, and the organism may have reduced the nitrates to nitrogen, ammonia, nitrous oxide, or hydroxylamine. Zinc ions reduce residual nitrates in the media to nitrites. A red color produced after addition of zinc indicates the presence of residual nitrates, confirming a true negative reaction. If a red or pink color does not occur after adding zinc, then the organism reduced the nitrate to a product other than nitrite, and the test is considered positive.

Which reagent(s) is (are) used to develop the red color indicative of a positive reaction in the nitrate reduction test? A. Sulfanilic acid and α-naphthylamine B. Ehrlich's and Kovac's reagents C. o-Nitrophenyl-β-D-galactopyranoside D. Kovac's reagent

D K. oxytoca and K. pneumoniae are almost identical biochemically except for the ability to produce indole. Both organisms are usually positive for urease, sucrose, and citrate. However, K. oxytoca is indole positive and K. pneumoniae is indole negative.

Which single test best separates Klebsiella oxytoca from K. pneumoniae? A. Urease B. Sucrose C. Citrate D. Indole

D The Shigella spp. are lactose nonfermenters that for the most part are biochemically inert and are classified into serogroups A, B, C, and D as a result of their biochemical similarity. S. sonnei is the species most often isolated from diarrhea cases in the United States. It is more active biochemically than the other species owing to ornithine decarboxylase and β-galactosidase activity. These enzymes, found in most strains of S. sonnei, distinguish it from other Shigella species.

Which species of Shigella is most commonly associated with diarrheal disease in the United States? A. S. dysenteriae B. S. flexneri C. S. boydii D. S. sonnei

A Acinetobacter spp. are nonmotile rods that appear as coccobacillary forms from clinical specimens. All are oxidase negative and catalase positive. P. aeruginosa reduces NO3 to NO2, while A. baumannii does not.

Which test group best differentiates Acinetobacter baumannii from P. aeruginosa? A. Oxidase, motility, NO3 reduction B. MacConkey growth, 37°C growth, catalase C. Blood agar growth, oxidase, catalase D. Oxidase, TSI, MacConkey growth

3. D Both organisms are oxidase positive, motile, and produce pyoverdin. Both are negative for ONPG and DNase. The differentiating tests are: Test P. aeruginosa P. putida Mannitol + Neg Reduce NO3 + Neg to NO2 42°C growth + Neg

Which tests are most appropriate to differentiate between Pseudomonas aeruginosa and Pseudomonas putida? A. Oxidase, motility, pyoverdin B. Oxidase, motility, lactose C. Oxidase, ONPG, DNase D. Mannitol, nitrate reduction, growth at 42°C

C (XLD agar is selective for gram-negative coliforms because of a high concentration (0.25%) of deoxycholate, which inhibits gram-positive bacteria. In addition, XLD is differential for Shigella and Salmonella spp. The medium contains xylose, lactose, and sucrose, which are fermented by most normal intestinal coliforms producing yellow colonies. Shigella does not ferment the sugars and produces red (or clear) colonies. Salmonella spp. ferment xylose; however, they also decarboxylate lysine in the medium, causing production of ammonia. Therefore, Salmonella first appear yellow but become red. Some Salmonella produce hydrogen sulfide (H2S) from sodium thiosulfate and therefore appear as red colonies with black centers.)

Xylose lysine deoxycholate (XLD) agar is a highly selective medium used for the recovery of which bacteria? A. Staphylococcus spp. from normal flora B. Yersinia spp. that do not grow on Hektoen agar C. Enterobacteriaceae from gastrointestinal specimens D. Streptococcus spp. from stool cultures


Related study sets

EXAM 4--CH. 17 (Integrated Marketing Communications)

View Set

Necessary versus Sufficient Conditions

View Set

Database Management I: Section 1-3 Test

View Set

Legal Environment of Business Final

View Set

Chapter 10: Fluid and Electrolytes

View Set

anatomy & physiology - muscular system gross anatomy

View Set

The Art of Public Speaking (Lucas) Chapter 9

View Set

Proof of Advanced Defensive Driver Course New Jersey

View Set